You are on page 1of 71

RT STUDENT CRT ENTRY EXAM REVIEW QUESTIONS

1.You have finished charting on your patient when you notice that an error was made. You should do which of the following? A.Tell the nurse so that she will tell the physician. B.Tell the nurse so that she will chart the correct information. C.Place a line through the error, initial it, and write in the correct information. D.Have your supervisor chart the correct information. C. From a legal point of view, a charting error should never be obliterated. Simply correct and initial the correct information. Only the person making the error should correct it. 2.In listening to a patient's lungs, you notice bronchial breath sounds in her right lower lobe. These would indicate which of the following? A.Normal lungs B.Pneumothorax C.Consolidation in her right lower lobe D.Pleural effusion in her right lower lobe C. Bronchial breath sounds are not normal in the right lower lobe and indicate consolidation of the alveoli. Neither pneumothorax nor pleural effusion can be identified by bronchial breath sounds. 3.You are called to start a new aerosolized medication treatment. After reading the physician's order, you notice that the drug dosage is outside the normal department guidelines. You should do which of the following? A.Give the treatment as ordered.

B.Contact the physician to confirm that the order is indeed correct. C.Give the treatment as ordered, and leave a note in the chart asking for clarification for the next treatment. D.Have the nurse rewrite the order. B. Never give an overdose of a medication. Personally contact the physician to clarify the error. (Rarely, a larger than usual dose of medication is given under special circumstances. The physician must make it clear that he or she is aware of the large dose and wants it delivered.)

4.You are called to the Emergency Department to help care for a patient who was in a car accident and has chest injuries, including broken ribs. While palpating her neck, you feel a crepitation. What is the most likely cause of this? A.She has a laryngeal tumor. B.Blood is in the back of her throat. C.She has aspirated a tooth. D.She has an air leak from her lung. D. The patient's history of injury and crepitus indicates air under the skin. The air under her skin would have to come from a lung tear. 5.You are called to help in the evaluation of a 55-year-old male patient. You notice the following signs and symptoms: oral temperature of 40 C (104.5 F), diaphoresis, respiratory rate of 22, the use of accessory muscles of respiration, and palpable rhonchi in the right lower lobe. You would suspect which of the following diagnoses: A.Bacterial pneumonia B.Heart attack

C.Pneumothorax D.Viral pneumonia A. All of the patient's symptoms point to a bacterial pneumonia problem. The other options may have some but not all of the noted symptoms. 6.Since being told of the diagnosis of cancer, a patient has become argumentative about his care and threatens to hit the nurse and therapist. He should be evaluated for A.Language barrier problems B.Hypercarbia C.Emotional state D.Hypoxemia C. The patient is reacting with anger to his diagnosis of cancer. He should be evaluated for emotional state. The other problems should not cause anger 7.To help determine your patient's level of consciousness, you should ask which of the following questions? 1.Do you know what day this is? 2.Can I see your identification wristband? 3.Do you know where you are? 4.How are you feeling today? 5.Do you know who the president is? A.2, 4 B.3 C.5 D.1, 3 A. The correct two questions relate to the patient's level of consciousness and understanding. The first question relates to the patient's understanding of time. The third question relates to the patient's understanding of place. The fifth question relates to the patient's understanding of person.

8.To help you determine whether your patient has orthopnea, you would ask which of the following? A.How many flights of stairs can you climb before you become short of breath? B.Do you know who the governor is? C.Do you need to use extra pillows behind your head and back to keep from getting short of breath when you sleep? D.Do any particular foods seem to make it harder for you to cough up your secretions? C. Orthopnea relates to the patient's inability to lie down and breathe comfortably. Extra pillows are needed to raise the head and body. The other questions relate to other areas of assessment. 9.In observing an infant's chest configuration, you notice that it is the same size in both the AP and lateral dimensions. This would indicate that the patient has A.A normal chest B.Funnel chest/pectus excavatum C.Pulmonary emphysema with air trapping D.Lordosis A. An infant's chest is basically round in dimension. 10.In examining your patient, you notice that she has greatly diminished breath sounds in her right lower lobe, and her trachea has shifted to the right. These signs indicate which condition? A.Right-sided pneumothorax B.Right-sided atelectasis C.Left-sided pneumothorax D.Left-sided pneumonia B. Diminished breath sounds mean that less air than normal is entering an area. A tracheal shift to the side of the diminished

breath sound indicates less air in the lung. Both of these point to atelectasis. 11.In palpating your patient for symmetrical chest movements, you notice that his left side does not move as much as his right side. This indicates that he has which condition or conditions? 1.Emphysema 2.Congestive heart failure 3.Left-sided pneumonia 4.Left-sided pneumothorax 5.Right-sided pneumonia A.1, 2 B.3, 4 C.4, 5 D.2 B. Left-sided pneumonia and pneumothorax both result in decreased movement on that side. Emphysema and congestive heart failure would not cause a one-sided change in movement. Right-sided pneumonia would result in less movement on the right side. 12.You are called to the Emergency Department to help evaluate a pediatric patient. On entering the room, you observe the patient's breathing effort and can hear a harsh, high-pitched sound on inspiration. Which of the following is true? A.Sounds are tracheal and normal. B.Sounds are bronchovesicular and not normal. C.Sounds are stridorous and indicate a respiratory emergency. D.Sounds are bronchial and indicate a respiratory emergency. C. Inspiratory stridor is the only listed breath sound that can be heard with the unaided ear. It is a respiratory emergency.

13.You are called to evaluate a patient's breathing pattern. You notice that the patient's tidal volumes go from small to large to small and then stop for 10 seconds before starting up again. The pattern repeats itself. This patient's breathing pattern would best be called A.Eupnea B.Obstructed expiration C.Kussmaul's respiration D.Cheyne-Stokes respiration D. Only Cheyne-Stokes respiration fits the description. 14.A tension pneumothorax is identified by which of the following? 1.Chest x-ray film shows a shift of the mediastinum toward the affected lung. 2.Chest x-ray film shows elevation of the hemidiaphragm on the affected side. 3.The patient's vital signs suddenly deteriorate. 4.Chest x-ray film shows a depression of the hemidiaphragm on the affected side. 5.Chest x-ray film shows a shift of the mediastinum away from the affected lung. 6.Vital signs are essentially unchanged. A.1, 2, 6 B.3, 4, 5 C.1, 2, 3 D.1, 3, 4 B. The high air pressure found with a tension pneumothorax causes the mediastinal contents to be shifted to the opposite side and the diaphragm on the affected side to be depressed. These drastic changes can cause the patient's vital signs to deteriorate rapidly. 15.Your patient is complaining of localized pain over the lower right area of the chest while breathing. When you auscultate her

chest, you hear a rasping noise at her point of pain on both inspiration and expiration. This is most likely A.Pleural friction rub B.Normal breath sounds C.Wheeze D.Rhonchi A. A pleural friction rub is identified as a localized area of abnormal grating breath sound; it is often localized to an area of pain on breathing 16.Your patient has distended external jugular veins, even though her head and body are raised 45 degrees above her legs. This would indicate that she A.Is hypertensive B.Is fluid overloaded C.Has emphysema D.Is dehydrated B. Fluid overload causes the jugular veins to be distended. Dehydration may result in the jugular veins being flat. Emphysema and hypertension should not have any effect on the jugular veins. 17.Tactile fremitus would be reduced in all of the following conditions EXCEPT A.Pneumothorax B.COPD C.Pulmonary edema D.Pleural effusion C. Tactile fremitus would be reduced in pneumothorax and COPD because the lung is overinflated. A pleural effusion would block and decrease the sounds coming from the lung. 18.A frail, thin patient known to have lung cancer is admitted to the hospital. His family

members are also present. What should be asked of them to make sure the proper level of care is delivered? A.The last time he ate B.The last time he had a bowel movement C.Whether any advance directives have been documented D.Whether he has brought his home care medications with him C. It is appropriate to ask about advance directives, such as a DNR order, for a patient with a fatal illness. Eating and bowel habits are not essential to know at this time. He will be given new orders for medications during his stay in the hospital, so it does not matter if he brought his medications with him. 19.It is most important to ask a patient with a broken ankle from a recent slip on an icy sidewalk about which of the following? A.Level of pain B.Level of consciousness C.Work of breathing D.Emotional state A. It is important to assess the patient's level of pain from the broken ankle. Severe pain should be managed with increased medication. The other issues are less important to assess unless there is an apparent problem. 20.All of the following could result in a mediastinal shift on a chest x-ray film EXCEPT A.Right-sided hemothorax B.Bilateral lower lobe pneumonia C.Left-sided tension pneumothorax D.Right lower lobe atelectasis B. All listed conditions except bilateral lower lobe pneumonia would shift the

mediastinum. Because bilateral lower lobe pneumonia affects both lungs, the mediastinum would stay centered properly. 21.A patient who is suffering respiratory distress would exhibit all of the following EXCEPT A.Normal respiratory rate B.Nasal flaring C.Intercostal retractions D.Use of accessory muscles of inspiration A. It is most likely that a patient with respiratory distress would show an increased respiratory rate 1.A hospitalized patient who recovered from a Clostridium botulinum infection received several respiratory care services. How should a nondisposable plastic pulmonary function testing mouthpiece be sterilized before being reused? A.Steam autoclave for 15 minutes. B.Soak in glutaraldehyde solution for 10 hours. C.Pasteurize for 20 minutes. D.Soak in an alcohol solution for 15 minutes. B. A glutaraldehyde solution soak for 10 hours is the only way listed to sterilize plastic equipment without damaging it. Putting plastic equipment into a steam autoclave causes it to melt and be destroyed. Pasteurization or soaking the equipment in an alcohol solution disinfects but does not sterilize the equipment as needed. 2.Select all of the following that can be used in the home to reduce the chance of bacterial growth in a small-volume nebulizer or other respiratory care

1.Soak the equipment in white vinegar. 2.Put the equipment in the oven, and turn on the broiler for 10 minutes. 3.Rinse a nebulizer in salt water (normal saline) after each use. 4.Wash the equipment in hot water with a detergent. A.1, 4 B.1, 2 C.1, 3, 4 D.1, 2, 3, 4 A. A hot water wash with detergent will remove secretions and other debris from the equipment. Soaking the equipment in white vinegar (acetic acid) will kill many of the microorganisms commonly found on home care equipment. The heat generated in an oven under the broiler will melt plastic used in a small-volume nebulizer and other respiratory care equipment. Rinsing the equipment in salt water will not make a significant difference in killing microorganisms. In addition, the dried salt crystals may plug up the capillary tube of a nebulizer. 3.After use of a mechanical ventilator has been discontinued, what is the best method to sterilize the reusable main-flow bacteria filter? A.Wrapping it and soaking it in acetic acid B.Pasteurization C.Glutaraldehyde soak D.Steam autoclaving D. Steam autoclaving is the only method listed that is acceptable for sterilizing the bacteria filter of a ventilator. Glutaraldehyde solutions will damage the filter medium and will make the filter useless. 4.A batch of respiratory care equipment has

gone through the gas sterilization process with ethylene oxide. Routine surveillance of the equipment shows that spores of Bacillus subtilis have survived the process. What should be done next? A.Use the equipment because this organism does not cause illness. B.Aerate the gas as usual, and put into use. C.Resterilize the equipment, and check for destruction of the spores. D.Wipe off the equipment with 70% alcohol to remove the spores from the equipment. C. The equipment must be resterilized because living spores of Bacillus subtilis indicate that the sterilization process was not successful. Other microbes also may have survived. Because of this concern, the equipment should not be used. Aeration of the equipment will not sterilize it, and 70% alcohol will not remove the spores from the equipment or sterilize it. 5.A retired home care patient living on a fixed income needs to be able to disinfect her respiratory therapy equipment. Which of the following would be best for her? A.Acetic acid B.Acid glutaraldehyde C.Ethylene oxide system D.Warm, soapy water A. Acetic acid is found in white vinegar. It is inexpensive and available in any grocery store. Realize that acetic acid is not a powerful disinfectant and does not kill most pulmonary pathogens. Acid glutaraldehyde and ethylene oxide systems are too expensive to use in a home. Warm, soapy water can be used to clean secretions from equipment but does not disinfect it. 6.What is the most cost-effective way for a respiratory care department to disinfect large

amounts of reusable plastic tubing and oxygen masks? A.70% ethyl alcohol B.Steam autoclave C.Pasteurization D.Dry heat C. Pasteurization is the least expensive way to disinfect large amounts of the plastic equipment used in respiratory care departments. Steam autoclave and dry heat are used to sterilize, not disinfect, equipment. In addition, the high temperatures used with these methods melt the plastic tubing and oxygen masks. Seventy percent ethyl alcohol is used only to wipe off the surfaces of large equipment items for disinfection. 7.A contaminated Bird Mark 7 intermittent positive-pressure breathing (IPPB) unit must be sterilized before use with another patient. What is the best method? A.Pasteurization B.Ethylene oxide C.10-Hour soak in glutaraldehyde D.Steam autoclave B. Ethylene oxide gas should be used on an IPPB machine because it sterilizes the unit without causing any damage. Pasteurization and glutaraldehyde involve solutions that damage the internal structures of any IPPB machine. The heat of steam autoclaving melts any plastic or rubber components of the unit and seriously damages the machine.

8.A 58-year-old patient had an exacerbation of his COPD related to spring allergies. As he is being prepared for discharge from the hospital, what should the respiratory therapist recommend? A.Get an influenza vaccination as soon as possible.

B.Get a tuberculosis skin test as soon as possible. C.Have a throat swab performed to check on a possible Streptococcus infection. D.Get an influenza vaccination in the fall. D. It is recommended that patients with cardiopulmonary conditions, like COPD, get an influenza vaccination each fall. In patients with preexisting conditions, the flu can be a very serious illness. Routine vaccination is also recommended for those older than 50 years of age. The flu is a seasonal problem through the fall and winter months. So, the vaccination should be given in September or October. There is no indication that the patient has been exposed to tuberculosis or has a throat infection. 1.Before drawing a blood gas sample from the radial artery, you should perform which test of adequate perfusion? A.Allen's test B.Modified Allen's test C.Blood pressure measurement D.P(A-a)O2 B. A modified Allen's test is used to determine whether adequate perfusion exists through the ulnar artery in case the radial artery should become occluded. This would ensure that the hand is still well perfused. The Allen's test is used to determine adequate perfusion through the radial artery. Adequate arm blood pressure does not ensure adequate perfusion of the hand, should the radial artery become blocked. The P(A-a)O2 test does not measure local perfusion. 2.A patient is brought into the emergency department after being rescued from a house fire. She is unconscious and has facial burns. The physician believes that she is suffering from smoke inhalation. What would you

recommend as the best way to evaluate her? A.ABGs analyzed through a CO-oximeter B.Pulse oximetry C.ABGs analyzed through a standard blood gas analyzer D.PtCO2 monitor A. Blood gas analysis through a COoximeter gives the most accurate results in that the unit can detect COHb. The other three choices cannot measure COHb level. Additionally, pulse oximetry and PtCO2 monitoring do not give CO2 or pH values. 3.You are ordered to draw a blood sample from your patient's radial artery. You test for adequate circulation by having the patient make a fist while you put pressure 1.Increased PaO2 2.Increased PaCO2 3.Decreased PaO2 4.Decreased PaCO2 5.Increased pH 6.Decreased pH A.1, 2, 6 B.3, 4, 5 C.2, 3, 6 D.3, 4, 6 C. The described test and its results are of a positive modified Allen's test. This positive result means that the patient has adequate ulnar circulation. 4. You are working in the intensive care unit when you notice that an arterial blood sample has been sitting out for 40 min. It was not put in ice-water. You could expect the blood gas analysis to be affected in which ways? 1. Increased PaO2

2. 3. 4. 5. 6.

Increased PaCO2 Decreased PaO2 Decreased PaCO2 Increased pH Decreased pH A. 1, 2, 6 B. 3, 4, 5 C. 2, 3, 6 D. 3, 4, 6

D. All of the listed conditions would warrant a blood gas analysis because they all deal with significant oxygenation and/or CO2 removal issues. 6.Safety guidelines for the protection of the therapist who is drawing an ABG sample include which of the following? 1.Put a glove on the hand used to draw the sample. 2.Put a glove on the hand with which you feel the pulse. 3.Put gloves on both hands. 4.Wear goggles. A.3, 4 B.1, 4 C.2 D.3 A. Standard Precautions necessitate that gloves be worn on both hands when blood may be contacted by either hand. Additionally, the eyes should be protected from possible blood splashes. 7.A 50-year-old patient has a PaO2 value of 72 torr when breathing room air. You would interpret this as A.Normal for a person of his age B.Mild hypoxemia C.Moderate hypoxemia D.Severe hypoxemia B. Review Table 3-3 on page 87 for the categories of hypoxemia 8.An acute rise in PaCO2 level from 40 to 50 torr would result in the following change in pH: A.Rise of 0.10 unit B.Fall of 0.05 unit C.Fall of 0.10 unit D.Rise of 0.05 unit

C. When a blood sample is not quickly cooled in ice-water, the living tissue will continue to consume O2 and produce CO2. The increased CO2 level will decrease the pH value. 5.You would recommend an arterial puncture to obtain a sample for blood gas analysis under which of the following conditions? 1.To measure the patient's PaO2 level after a change in the patient's inspired O2 concentration 2.Suspected CO poisoning 3.To measure the patient's PaCO2 level after a change in the patient's minute volume 4.After the patient has been admitted to the emergency department with a tension pneumothorax 5.During a cardiopulmonary resuscitation attempt A.1, 2 B.1, 3 C.2, 3, 4 D.1, 2, 3, 4

B. Shapiro and associates (1994) have stated that an acute rise in CO2 of 20 mm Hg results in a drop in pH of 0.10 unit. So, an acute rise in CO2 of 10 mm Hg would result in a drop in pH of 0.05 unit. 9.Interpret the following blood gas drawn from a patient who is breathing 40% O2: PaO2, 54 torr; SaO2, 87%; pH, 7.37; PaCO2, 62 torr; bicarbonate, 38 mEq/L; and base excess, +11 mEq/L. 1.Corrected hypoxemia 2.Uncorrected hypoxemia 3.Metabolic alkalosis 4.Uncompensated metabolic acidosis 5.Compensated respiratory acidosis A.1, 4 B.1, 3 C.2, 4 D.2, 5 D. A PaO2 value of less than 80 torr is uncorrected hypoxemia. A compensated respiratory acidosis is indicated by the increased PaCO2 value coupled with an increased bicarbonate level and increased base excess found with a normal pH. (Review Table 3-2 on page 87 and Table 3-8 on page 90.) 10.Interpret the following blood gas drawn from a patient who is breathing 35% O2: PaO2, 86 torr; SaO2, 90%; pH, 7.29; PaCO2, 37 torr; bicarbonate, 17 mEq/L; and base excess, 8 mEq/L. 1.Corrected hypoxemia 2.Uncorrected hypoxemia 3.Compensated metabolic acidosis 4.Uncompensated metabolic acidosis 5.Compensated respiratory acidosis A.2, 4 B.1, 4

C.2, 5 D.1, 3 B. A PaO2 value greater than 80 torr with supplemental O2 is corrected hypoxemia. An uncompensated metabolic acidosis is indicated by the normal PaCO2 value coupled with decreased bicarbonate concentration and decreased base excess found with an acidotic pH. (Review Table 32 on page 87 and Table 3-8 on page 90.)

11.Interpret the following blood gas drawn from a patient who is breathing 21% O2: PaO2, 117 torr; SaO2, 98%; pH, 7.57; PaCO2, 20 torr; bicarbonate, 24 mEq/L; and base excess, +1 mEq/L. 1.Normal oxygenation 2.Excessively corrected hypoxemia 3.Uncompensated respiratory alkalosis 4.Uncompensated metabolic acidosis 5.Compensated respiratory and metabolic alkalosis A.2, 3 B.2, 4 C.1, 3 D.1, 4 C. Normal oxygenation is indicated because the patient's PaO2 level is elevated as a result of hyperventilation (PaCO2 value of 20 torr). An uncompensated respiratory alkalosis is indicated by the low PaCO2 value coupled with normal bicarbonate concentration and normal base excess found with an alkalotic pH. (Review Table 3-2 on page 87 and Table 3-8 on page 90.) 12.Interpret the following blood gas drawn from a patient who is breathing 60% O2: PaO2, 72 torr; SaO2, 94%; pH, 7.18; PaCO2, 50 torr; bicarbonate, 18 mEq/L; and

base excess, 10 mEq/L.

decreased base excess found with a normal pH. (Review Table 3-2 on page 87 and Table 3-8 on page 90.) 14.Which of the following would indicate that a patient's tissues are adequately oxygenated? A.PaO2, 85 torr B.PO2, 30 torr C.SO2, 75% D.SaO2, 90% C. An S O2 value of 75% is normal and correlates with normal tissue oxygenation. A P O2 value of 30 torr indicates tissue hypoxia. Normal ABG values do not necessarily correspond to normal tissue oxygenation values. (See Table 3-11 on page 91.) 15.Blood gas analyzer calibration values are considered to be in control if they are A.Within 1 SD of the norm B.Within 2 SDs of the norm C.Within 3 SDs of the norm D.Within 4 SDs of the norm B. Two SDs are considered in control with a blood gas analyzer. (See Figure 3-8 on page 90.) 16.A 50-year-old patient with emphysema seems to be tiring after 30 min into a weaning attempt on a Briggs adapter (T piece). The best way to evaluate her ventilatory status is by A.Checking her PaO2 value B.Measuring a PtcCO2 value C.Checking her PaCO2 value D.Performing a bedside vital capacity measurement C. The patient's PaCO2 value best correlates with her ventilatory status and level of fatigue. The other tests are of value but give

1.Uncorrected hypoxemia 2.Corrected hypoxemia 3.Uncorrected respiratory acidosis 4.Uncorrected metabolic acidosis 5.Combined metabolic and respiratory acidosis A.1, 5 B.2, 5 C.2, 3 D.2, 4 A. A PaO2 level of less than 80 torr is uncorrected hypoxemia. A combined metabolic and respiratory acidosis is indicated by the increased PaCO2 value coupled with decreased bicarbonate level and decreased base excess found with an acidotic pH. (Review Table 3-2 on page 87 and Table 3-8 on page 90.) 13.Interpret the following blood gas drawn from a patient who is breathing 24% O2: PaO2, 57 torr; SaO2, 91%; pH, 7.45; PaCO2, 22 torr; bicarbonate, 16 mEq/L; and base excess, 6 mEq/L. 1.Corrected hypoxemia 2.Uncorrected hypoxemia 3.Compensated respiratory alkalosis 4.Uncompensated respiratory alkalosis 5.Combined metabolic and respiratory acidosis A.1, 3 B.1, 4 C.2, 3 D.2, 5 C. A PaO2 value of less than 80 torr is uncorrected hypoxemia. A compensated respiratory alkalosis is indicated by the decreased PaCO2 value coupled with decreased bicarbonate concentration and

less direct evidence of her ability to breathe effectively. 17.Your patient has Guillain-Barr syndrome and pneumonia. The patient has just been placed on 35% O2 by mask. The physician asks for your suggestion on the best way to evaluate the patient's overall ability to breathe. You would recommend A.Doing a full set of pulmonary function tests B.Drawing an arterial blood sample for analysis C.Performing pulse oximetry D.Performing pulse oximetry and a force vital capacity measurement A.63 torr B.75 torr C.54 torr D.105 torr B. The results of an ABG measurement inform you of the patient's oxygenation status and PaCO2 level. Pulse oximetry gives information only on oxygenation status. Pulmonary function tests do not give any information on the patient's PaO2 and PaCO2 levels. Additionally, a full set of pulmonary function tests can be very tiring for the patient. 18. You are called to evaluate a patient who is using a pulse oximeter. Upon entering the room, you notice a black woman with an oximeter probe on her right earlobe. The monitor shows a weak pulse signal and a fluctuating SpO2 value. Which of the following would you do in an attempt to correct the problem? 1. Try monitoring from a fingertip.

3. Cover the probe with an opaque wrap. 4. Switch the probe to the left earlobe. A. 2 B. 1, 3 C. 2, 4 D. 3 B. Some pulse oximeters obtain inaccurate readings through the skin of darkly pigmented patients. Blocking outside light with an opaque wrap or moving the probe to a lightly pigmented area such as a fingertip (without nail polish) often results in accurate readings. (See Table 3-12 on page 94.) 19. You are working with a postanesthesia patient who is on a PtcCO2 monitor. The correlation factor between the PaCO2 and PtcCO2 is 1.4. The patient's previous PtcCO2 level was 63 torr. The nurse has called you because it is now 75 torr. The patient's approximate PaCO2 value would be calculated as A. 63 torr B. 75 torr C. 54 torr D. 105 torr C. The patient's approximate PaCO2 value of 54 torr is found by dividing the PtcCO2 value of 63 torr by the correlation factor of 1.4. (See page 100 for an example calculation.) 20.A 35-year-old patient with pneumonia is receiving mechanical ventilation with positive end-expiratory pressure (PEEP).

2. Switch to a probe over the bridge of the nose.

You are ordered to calculate and interpret the patient's P(A-a)O2 level. The following conditions exist: PB = 750 torr. Normal is 760 torr for sea level. PH2O = 54 torr because your patient's temperature is 104 F/40C. Normal is 47 torr for a normal temperature. FiO2 = 0.5 for 50% inspired oxygen. Normal is 0.21 for room air. PaCO2=36torr PaO2=60torr RespiratoryExchangeRatio=0.8 PAo2=[(PB-PH2O)Fio2]PaCO2 0.8 Based on the listed conditions, what is the patient's PAO2 value? A.312 torr B.303 torr C.101 torr D.95 torr B. The patient's PAO2 value (pressure of alveolar oxygen) can be calculated as follows:

B. The patient's P(A-a)O2 value (difference between alveolar pressure and arterial pressure of oxygen) can be calculated as follows:

22.How should the patient's P(A-a)O2 results be interpreted? A.There is an error; check the blood gas analyzer. B.Normal oxygenation for 50% oxygen being inspired C.Normal for a patient of this age D.Larger than normal difference D. The normal P(A-a)O2 difference should be no more than 25 torr for a healthy person of this age. The patient's difference of 243 torr is far greater than normal. No indication is found that the blood gas analyzer needs recalibration. If a normal person was breathing 50% oxygen, the PaO2 value would be much higher (about 275 torr) than this patient's PaO2 value. The difference is much larger than normal for a patient of this age. (See Figure 3-17 on page 101.) 1.Which of the following statements is/are true of the MEP test? 1.A pressure of 20 to 25 cm H2O usually is adequate. 2.A pressure of +20 to +25 cm H2O usually is adequate. 3.A pressure of +40 cm H2O is usually adequate. 4.It is a good indicator of the patient's ability to cough. 5.The patient should hold the effort for 1 to 3 seconds.

21.Based on the listed conditions, what is the patient's P(A-a)O2 value? A.248 torr B.243 torr C.232 torr D.41 torr

A.1, 2 B.3, 4 C.3 D.3, 4, 5 D. The MEP test is a measure of expiratory muscle strength. A value of +40 cm H2O usually indicates enough strength for spontaneous breathing and the ability to cough effectively. The pressure should be sustained for a short time to ensure that the measurement is accurate. 2.The predicted FVC value for African Americans is A.10% to 15% higher than that for Caucasians B.The same as for Caucasians C.10% to 15% less than that for Caucasians D.20% to 25% less than that for Caucasians C. African Americans have been shown to have an FVC that is 10% to 15% less than age- and height-matched Caucasians. Review the FVC discussion. 3.Which of the following test results is/are needed to calculate TLC? 1.FRC 2.RV 3.VT 4.ERV 5.IC 6.VC A.1, 3 B.2, 6 C.2, 5 D.1 and 5, or 2 and 6 D. Review Figure 4-14 for volumes and capacities 4.A normal MEFV loop test would show

A.FEF50% less than FIF50% B.Predicted lung diffusion ability C.FEF50% greater than FIF50% D.A normal FRC A. Flow at the midpoint of expiration normally is less than flow at the midpoint of inspiration, because the small airways are beginning to close at the halfway point of an expiratory effort. A flow-volume loop test cannot measure lung diffusion or RV to calculate the FRC 5.A patient with a neuromuscular disease has been having serial bedside spirometry performed. Over the past 4 hours, her VC and MIP values have been decreasing. How should this be interpreted? A.Her strength is improving. B.She is not performing to the best of her ability. C.She has undiagnosed asthma. D.Her condition is worsening. D. A progressively declining VC and MIP indicate that the patient is getting weaker and her condition is getting worse. If her condition were improving, these tests would show increasing values. It is up to the respiratory therapist to ensure that the patient is performing the tests properly. If done properly, the test results will be valid. The MIP test is not used to help in the diagnosis of asthma, because it measures strength, not flow. 6.An order is received to perform the following bedside spirometry tests on a patient: tidal volume, FVC, and peak flow. Which device would you take with you to perform the tests? A.Stead-Wells water-seal spirometer B.Maximum inspiratory pressure manometer C.Differential pressure pneumotachometer D.Body plethysmograph

C. A differential-pressure pneumotachometer is portable enough to be taken to a patient's bedside. It can be used to determine a patient's tidal volume, FVC, and peak flow. A Stead-Wells water-seal spirometer is not typically moved, because it is large, and the water will be splashed about while it is moving. The device can be used for all three listed tests when done in the pulmonary function laboratory. A maximum inspiratory pressure manometer is used for the maximum inspiratory pressure (MIP) test. It may be reconfigured for a maximum expiratory pressure (MEP) test but cannot measure any gas flows. A body plethysmograph is far too large to move to a patient's bedside. However, it can be used for all three listed tests when they are done in the pulmonary function laboratory. 7.Before a patient does an FVC test, the pneumotachometer should have the following done: A.The gas analyzer should be calibrated. B.A CO2-absorbing material should be placed in line with the circuit. C.A 3-L volume should be pumped into and out of the circuit to check for leaks. D.The kymograph speeds should be checked. C. CO2 buildup is not a problem with the FVC test, because the patient does not rebreathe his or her own gas for very long. A gas analyzer and kymograph are not used with an FVC test. 8.To help in the diagnosis of a patient with a questionable history of wheezing and possible asthma, which of the following would be the best test? A.Bronchoprovocation study B.Flow-volume loop C.Before and after bronchodilator study D.Raw

A. A bronchoprovocation study would be the best test for a patient with an unclear history or signs and symptoms of asthma. If the patient has any airway hypersensitivity, this test will cause a measurable decrease in expiratory flow. The other three tests can be used in the diagnosis of asthma but are not as specific to asthma as a bronchoprovocation study. 9.A patient has just been tested for CL in a body plethysmograph. Her compliance was determined to be 0.2 L (200 mL)/cm H2O. Based on this, she most likely has A.Asthma B.Pulmonary fibrosis C.Emphysema D.Normal lungs C. Emphysema is the only condition listed in which the CL would be increased. Normal compliance is 0.1 L (100 mL)/cm H2O. 10.Calculate a patient's inspiratory time and expiratory time when he has an I : E ratio of 2 : 1 and a respiratory rate of 15/min. A.2.7 sec for inspiration and 1.3 sec for expiration B.3.3 sec for inspiration and 1.7 sec for expiration C.1.3 sec for inspiration and 2.7 sec for expiration D.1.7 sec for inspiration and 3.3 sec for expiration A. Review the calculation on page 112. 11.When a patient performs an MEP test, it is important that he or she A.Blow out all air before starting the effort B.Breathe in a VT and blow out hard C.Inhale to TLC and blow out hard D.Exhale a VT breath and inhale as hard as possible.

C. It has been determined that blowing out from TLC results in the greatest expiratory force. This is the recommended procedure. 12.A patient weighs 45 kg (100 lb). Her predicted VT would be A.550 mL B.450 mL C.350 mL D.250 mL C. VT is estimated as 3 to 4 mL/lb (or 7 to 9 mL/kg) of ideal body weight, which results in a range of about 300 to 400 mL. 13.You receive an order to calculate your patient's alveolar ventilation. His respiratory rate is 16, and his average VT is 580 mL. He weighs 170 lb. His alveolar ventilation is A.2720 mL B.410 mL C.750 mL D.510 mL B. Alveolar ventilation is estimated by subtracting the patient's ideal body weight in pounds (170) from the VT (580 mL). 14.Your patient has an FEV1% that is calculated to be 80% of his FVC. On the basis of this finding, the patient probably A.Is having an asthma attack B.Has a laryngeal tumor C.Has a fibrotic lung disease D.Is clinically normal D. The patient exhaled a normal percentage of his or her VC in 1 second. The other conditions would all result in a low value. Review the normal values discussed earlier in the chapter. Note: Refer to the figure on p. 147 for questions 15 and 16.

15.Which section of the spirometry tracing represents the FVC? A.2 B.3 C.4 D.5 B. The FVC is the largest capacity that can be measured by Spirometry 16.Which section of the spirometry tracing represents the VT? A.1 B.2 C.3 D.4 A. The VT would be seen as the repeated smallest volumes. 17.The VC is made up of 1.RV 2.FRC 3.ERV 4.VT 5.IRV A.2, 4 B.3, 4 C.1, 2, 3 D.3, 4, 5

D. The VC is made up of these three volumes. 18.Which of the following are true of the PF measurement? 1.It is usually seen at the end of the patient's FVC effort. 2.It increases with height. 3.It increases with age. 4.It decreases with age. 5.It is usually seen at the beginning of the patient's FVC effort. A.1, 2, 3 B.2, 4, 5 C.1, 2, 4 D.4, 5 B. PF is directly related to height and indirectly related to age. When done properly, the PF will be seen at the start of an FVC effort, because that is when air is emptied from the upper airway 19.You are having your patient perform the MIP test. His three attempts produce these results: 15 cm H2O, 45 cm H2O, and 20 cm H2O. The best explanation for these values is that A.The patient is starting from the FRC. B.The equipment has a large leak. C.The patient is starting from the RV. D.The patient is not trying his best every time. D. Inconsistent high and low pressures are most likely the result of inconsistent effort. A leak would result in a consistently low value. A patient should have a consistent value starting from either the RV or the FRC. 20.The physician wants to know whether a new bronchodilator would be helpful to his patient with asthma. He orders a before and

after bronchodilator study. The patient has the following peak flow values: 7.5 L/min before the medication and 9.4 L/min after the medication. Calculate her percentage change.

A.25% B.1.25% C.25% D.80% A. Review the calculation on page 121 1.The waveform sequence seen during the insertion of a pulmonary artery catheter is A.RA, RV, PAP, PCWP B.RV, RA, PAP, PCWP C.RA, RV, PCWP, PAP D.Ao, RA, RV, PAP A. The pulmonary artery catheter first enters the right atrium (RA) of the heart. It then follows the flow of blood through the heart by entering the right ventricle (RV). Next the catheter enters the pulmonary artery and the pulmonary artery pressure (PAP) is recorded. When the balloon at the tip of the catheter advances as far into the pulmonary artery as it can, it wedges in place. The resulting pressure is the called the pulmonary capillary wedge pressure (PCWP). 2.When evaluating a patient's stroke volume, which of the following is true? A.It is an indicator of the adequacy of perfusion of the body tissues. B.It is the output of blood for 1 minute. C.It has a range of 50 to 120 mL in the adult. D.It is the resistance to flow. C. Stroke volume is the blood volume pumped by each ventricle with each heartbeat and has a range of 50 to 120 mL in

the adult. Cardiac index (CI) is an indicator of the adequacy of perfusion of the body's tissues. Cardiac output (CO) is the output of blood for 1 minute. Afterload is the resistance to flow. It is caused by the level of tone in the blood vessels and the viscosity of the blood. (See Box 5-2 on page 160.) 3.A patient with chronic bronchitis is being monitored with regular measurements of arterial blood gas values and capnometry. The following data are available: PaCO2 PaO2 PETCO2 PCO2 53 torr 67 torr 33 torr 20 torr

The patient's VD/VT or VD fraction can be recorded as 0.62 or 62%. (Note that this equation must be used when the patient's tidal volume is not known.) 4.A 40-year-old patient recovering from ARDS is receiving mechanical ventilation with a tidal volume of 650 mL. The patient has an arterial line, a pulmonary artery catheter, and capnometry for monitoring. The following information is gathered after a change in PEEP level:

Calculate the patient's VD/VT. A.0.30 B.0.38 C.0.62 D.0.71 C. This problem requires the calculation of the decimal fraction or percentage of dead space. To do so, place both the patient's arterial and end-tidal carbon dioxide values into this formula:

43 torr 79 torr 32 torr 38 torr 22 torr

PaCO2 PaO2 PO2 PETCO2 PCO2

Calculate the patient's VD. A.273 mL B.319 mL C.338 mL D.384 mL B. This problem requires the calculation of the patient's dead space volume. To do so, place both the patient's arterial and end-tidal carbon dioxide values into this formula:

In which:

Calculate the VD/VT as follows:

In which:

A.Decreased lung markings on chest radiograph B.Increased pulmonary capillary wedge pressure C.Increased PaO2 D.Decreased pulmonary artery pressure B. If excessive fluid is given to a patient with congestive heart failure, the fluid is likely to accumulate in the pulmonary vessels because the left ventricle cannot pump effectively. This backup of fluid into the lungs will increase the pulmonary capillary wedge pressure. The patient should be monitored for pulmonary edema. If the patient develops pulmonary edema, a chest radiograph will show increased, rather than decreased, lung markings and fluid in the lungs. The patient's PaO2 level can be expected to decrease rather than increase if pulmonary edema develops. If fluid builds up from the left ventricle into the pulmonary vessels, the pulmonary artery pressure will increase along with the pulmonary capillary wedge pressure. 7.A patient hospitalized with leg vein thrombosis experiences sudden shortness of breath. Which of the following should be recommended to evaluate the patient's situation? A.Lung compliance B.Electrocardiogram C.Chest radiograph D.VD/VT D. The patient's history and current situation suggest that a pulmonary embolism has developed. The VD/VT test should be performed to determine whether the patient's dead space has increased. If it has, this will match the clinical suspicion of a pulmonary embolism. It is doubtful that the patient's lung compliance will change if a pulmonary embolism occurred. There is no specific

Calculate the VD as follows:

The patient's physiologic VD volume equals 319 mL. (Note that this equation must be used when the patient's tidal volume is known.) 5.A 35-year-old patient in the intensive care unit has the following hemodynamic data. Which of these data indicates a problem with the patient? A.SVR of 600 dynes/sec/cm5 B.CI of 3 L/min/m2 of body surface area C.PO2 of 38 torr D.Shunt of 4% A. A normal adult's systemic vascular resistance (SVR) has a range of 900 to 1400 dynes/sec/cm5. This patient's SVR value of 600 dynes/sec/cm5 is well below normal. This may result from vasodilation or hypovolemia. All of the other values are within normal range. (See Box 5-2 on page 160.) 6.A patient with a history of congestive heart failure is inadvertently given intravenous fluids of 2000 mL instead of the ordered amount of 200 mL. Which of the following is most likely to be seen?

change in the electrocardiogram that corresponds with a pulmonary embolism or other sudden cause of shortness of breath. Chest radiograph changes are unlikely soon after a pulmonary embolism has occurred. 8.Capnography will be used to monitor a patient's recovery from anesthesia. What gas should be used for the zero calibration? A.Room air for 0% carbon dioxide B.Room air for 21% oxygen C.5% carbon dioxide D.The same concentration of anesthetic gas as used with the patient A. A capnograph is zero calibrated on room air because it contains only a trace of carbon dioxide. A capnograph cannot measure oxygen percentage or anesthetic gases. Either 5% or 10% carbon dioxide is often used for the second point (high point) in two-point calibration of the capnograph. 9.Hypovolemia in an adult patient would be indicated by a PCWP of A.2 mm Hg B.8 mm Hg C.12 mm Hg D.24 mm Hg A. A PCWP level of 2 mm Hg is very low. (See Box 5-2 on page 160.) 10.An adult patient has had a pulmonary artery catheter inserted. A normal PAP pressure in this patient would be A.8 mm Hg B.25/10 mm Hg C.35/15 mm Hg D.120/80 mm Hg B. The normal PAP level is about 25/10 mm Hg. The normal PCWP value is about 8 mm Hg. A PAP value of 30/15 mm Hg would be elevated. Normal arterial blood pressure is about 120/80 mm Hg. (Review the values in

Box 5-2 on page 160.) 11.The principle of operation of the capnography monitor is A.The same as that of the Clark electrode B.The same as that of the Severinghaus electrode C.Infrared absorption D.The same as that of the CO-oximeter C. Infrared absorption is the main operating principle for bedside capnography units. 12.Calculate a patient's pulmonary artery diastolicpulmonary capillary wedge pressure (PAd-PCWP) gradient if the PAP is 30/12 mm Hg and the PCWP is 8 mm Hg. A.38 mm Hg B.12 mm Hg C.8 mm Hg D.4 mm Hg D. The difference between the PAP diastolic pressure of 12 mm Hg and the PCWP pressure of 8 mm Hg is 4 mm Hg. 13.A patient has an end-tidal CO2 pressure of 30 torr and a P(a-et)CO2 gradient of 4 torr. The alveolar to end-tidal gradient is in the usual direction. Based on this, his PaCO2 would be estimated as A.4 torr B.26 torr C.30 torr D.34 torr D. The patient's estimated PaCO2 value is 34 torr. It is found by adding the PetCO2 value of 30 torr and the P(a-et)CO2 gradient value of 4 torr. Review the math examples in the chapter, if necessary 14.Your patient is known to have advanced COPD. When checking his VD/VT ratio, you would expect it to be

A.Unaffected by his condition B.Increased C.Normal D.Decreased B. Patients with COPD have significant ventilation-to-perfusion mismatches. This results in increased dead space ventilation and shuntlike effects. The COPD patient would not be expected to have normal or decreased dead space. 15.A patient is being mechanically ventilated and has a reflectance oximetry pulmonary artery catheter in place. What SO2 value would indicate the patient is oxygenating adequately? A.40% B.50% C.75% D.90% C. An S 2 level of 75% would be normal and indicate normal tissue oxygenation. Values of less than 75% saturation would indicate tissue hypoxia. A saturation of 90% is far higher than that needed in venous blood. 16.The normal range for the P(a-et)CO2 gradient is A.<1 torr B.1-5 torr C.More than 15 torr D.About 40 torr B. When ventilation and perfusion match well, as in a normal person, the gradient between the arterial carbon dioxide level and the exhaled carbon dioxide level is 2 to 3 torr with a range between 1 and 5 torr. A carbon dioxide level of less than this would indicate either no alveolar ventilation or no production of CO2. Gradients of greater than 5 torr have been found in patients with

significant ventilation/perfusion mismatching. 17.An adult patient has been admitted for observation after suffering a concussion in a fall. Her arterial blood pressure is found to be 115/78 mm Hg. How should these results be interpreted? A.Within normal limits B.Hypertension C.Hypotension D.She has an intracranial bleed. A. The patient's blood pressure of 115/78 mm Hg is within normal limits for an adult. Blood pressure cannot be used as an indicator of an intracranial bleed. (See Box 5-2 (Page 160) and review Chapter 1, if necessary.) 1.Your home care patient has a problem with his O2 concentrator and needs to change to his H-tank of O2. If his nasal cannula is receiving a flow of 3 L/min and the tank pressure is 1300 psig, how long can the patient receive O2? A.Longer than 22 hours B.Longer than 1300 hours C.2 hours D.Longer than 120 hours A. See the sample H-tank duration calculation given earlier in the chapter. Common errors include using the E-tank factor and failing to convert from minutes to hours by dividing by 60. 2.What is the most likely problem to watch for in a patient with severe COPD who is receiving supplemental O2? A.Pulmonary edema from O2 toxicity B.Hypoventilation C.Retinopathy of prematurity (ROP) D.Hyperventilation

B. A patient with COPD who is hypercarbic and is breathing on hypoxic drive should be given supplemental O2 with great care. Too much O2 will result in too high an arterial O2 level and will blunt the hypoxic drive. Hypoventilation will result in a rising CO2 level. Pulmonary edema from O2 toxicity would necessitate a high percentage of O2 for an extended period. ROP is seen only in premature infants. Hyperventilation is not caused by breathing of supplemental O2. 3.Which of the following is the best device for giving 35% oxygen to an alert 2-yearold? A.Oxygen tent B.Nasal cannula at 3 L/min C.Simple mask at 3 L/min D.Incubator A. An oxygen tent will provide 35% oxygen to an alert 2-year-old patient without restricting normal movement. It is doubtful that an alert, active 2-year-old will keep either a nasal cannula or a simple mask in place as needed. In addition, the actual oxygen percentage cannot be measured through these devices. This child is too large for an incubator. 4.You are making general rounds in the hospital when you find a patient whose reservoir tubing has fallen off his 40% Tpiece. This would result in which of the following? A.Increased inspired O2 A.Increased inspired O2 B.Increased inspired CO2 C.Decreased inspired CO2 D.Decreased inspired O2 D. The reservoir tubing holds O2 from which the patient can inspire. Losing the reservoir results in inhalation of room air and a

decrease in the overall O2 percentage. When set up properly, any exhaled CO2 is blown clear from the reservoir tubing before the next inspiration. 5.The risks of O2 therapy include all of the following EXCEPT A.Pulmonary O2 toxicity B.Denitrogen absorption atelectasis C.O2-induced hyperventilation D.Retinopathy of prematurity (ROP) C. No known condition of O2-induced hyperventilation exists. Do not confuse this with O2-induced hypoventilation, which needs to be monitored in some patients with COPD. See the rationale for Question 2. 6.Your patient is wearing a face tent because of recent facial surgery. It is set at 35% O2. The nurse moves the patient from an upright to a supine position in bed. What effect will this have on her respiratory status? A.Increased VT B.Increased inspired O2 C.Increased inspired CO2 D.Decreased inspired O2 D. Because the face tent is open on top and O2 is heavier than room air, the O2 in the face tent tends to pour out if the patient lies supine. When all is set up properly, CO2 should not build up in a face tent, no matter the patient's position. A face tent should not influence a patient's VT. Lying supine may result in decreased VT. 7.To minimize the risk of hypoxemia during a treatment or procedure, you would do which of the following? 1.Increase the O2 percentage by 20% above the normal setting before suctioning or changing equipment 2.Keep the O2 percentage the same as if the

patient is not hypoxemic at this time 3.Minimize the time that the patient would be breathing room air 4.Increase the O2 percentage to 100% before suctioning 5.Make sure the replacement equipment is working properly before you place it on the patient A.1, 3 B.3, 4, 5 C.2, 5 D.3, 4 B. The three listed items are all important for minimizing hypoxemia during a treatment or procedure. Increasing the O2 by 20% will help to minimize hypoxemia, but it is appropriate to raise the percentage much higher. 8.Initially, the O2 percentage found in the O2 tent of a 2-year-old child was found to be stable at the ordered 35%. Now, less than that is found. All of the following should be tried EXCEPT A.Add an additional flowmeter and run them both at flush B.Keep the tent canopy tightly tucked under the mattress C.Keep the canopy flaps closed when the child is not receiving nursing care D.Check the analyzer for proper function A. Running two flowmeters at flush would put too much O2 into the tent and probably would raise the percentage to a far greater extent than ordered. In addition, the noise level would be high, which could be harmful to the child's hearing. Closing any source of leaks would help keep the O2 level stable. The analyzer may be malfunctioning and may be reporting a low value. 9.Your patient is wearing a partialrebreathing mask. The reservoir bag almost

totally collapses during inspiration. You would do which of the following? A.Tell the patient to breathe more slowly B.Put a nasal cannula on the patient C.Tell the patient to breathe more rapidly D.Increase the O2 flow D. When properly operating, a partialrebreathing mask's reservoir bag should not collapse during inspiration. Raise the O2 flow so that the bag stays at least two-thirds full during inspiration. Hypoxic patients usually breathe in at whatever pattern and rate is most efficient for them. It may be counterproductive to try to have the patient breathe differently. A nasal cannula cannot deliver as high an O2 percentage as can be delivered by the partial-rebreathing mask. 10.When checking your home care patient's reservoir-type nasal cannula, you notice that the reservoir does not fill and empty in synchrony with the patient's breathing pattern. Based on this, you would A.Increase the O2 flow to deliver the intended amount B.Replace the cannula C.Decrease the O2 flow to unstick the reservoir membrane D.Switch the patient to an air entrainment mask at approximately the same O2 percentage as that given to the patient who received it by cannula B. The reservoir membrane is defective, so the cannula should be replaced. The O2 flow should not be increased because this would give the patient more O2 than is intended. Decreasing (or increasing) the O2 flow will not unstick a defective reservoir membrane. Changing to an air entrainment mask would be a possible remedy if a replacement cannula did not exist. 11.What O2 delivery device would you

recommend for a patient who has a variable respiratory rate, I : E ratio, and VT? A.Nasal cannula B.Air entrainment mask C.Simple O2 mask D.Face tent B. An air entrainment mask (high-flow delivery system) is designed to provide enough flow of the prescribed O2 percentage (fixed concentration) at any patient rate, VT, and so forth. The other three devices do not provide enough flow to deliver a consistent, known O2 percentage. 12.The physician asks you which O2 delivery device would be best for a patient who needs about 75% O2. You would recommend which of the following? A.Nonrebreathing mask B.Face tent C.Air entrainment mask D.Simple O2 mask A. A properly fitting nonrebreathing mask with enough flow to keep the reservoir bag inflated will deliver the highest O2 percentage of all available devices. 13.A patient has a nasal cannula and needs to be transported on a stretcher. The E-sized O2 cylinder will have to be laid flat under the stretcher. Which flowmeter would you recommend? A.Backpressure-compensated Thorpe B.Nonbackpressure-compensated Thorpe C.Bourdon D.Backpressure-compensated kinetic C. A Bourdon flowmeter is the only unit that will accurately indicate the flow when it is laid horizontally. The others read accurately only in a vertical position.

14.An E-cylinder of O2 needs to be prepared for transport of a patient. You would look for a regulator with which pinhole locations? A.1 and 5 B.2 and 6 C.3 and 5 D.2 and 5 D. Review the information on pinholes in Table 6-3 on page 201. 15.What is the duration of flow of an Ecylinder with 1700 psig that is running at 5 L/min? A.0.9 hour B.1.6 hours C.7.7 hours D.13.7 hours B. Review the sample E-tank duration calculation in this chapter. Common errors include using the H-tank factor and failure to convert from minutes to hours by dividing by 60. 16.You are called to draw an arterial blood sample from a patient who is wearing a 35% air entrainment mask. When you enter the room, you notice that his covers are drawn up over the air entrainment ports of the mask. How would this affect the function of the mask? A.The total flow will be increased. B.There will be no effect. C.The O2 percentage will be increased. D.The O2 percentage will be decreased. C. Covering the air entrainment ports on an air entrainment mask will result in the patient's receiving a higher O2 percentage than is desired. In addition, the total flow will be decreased. 17.You receive a call at the office from one

of your home care patients. She reports that the high-pressure pop-off valve on the bubble humidifier to her transtracheal oxygen catheter is venting. In addition, she cannot flush out the catheter with saline or push the cleaning rod through it. What should you tell her to do? A.Remove the humidifier and double the oxygen flow rate to the catheter. B.Force the saline through the catheter until the obstruction is cleared. C.Force the cleaning rod through the catheter until the obstruction is cleared. D.Switch her oxygen from the transtracheal catheter to a nasal cannula. D. Because a major obstruction exists in the transtracheal oxygen catheter, it should be replaced, but first the patient must switch her oxygen delivery system to a nasal cannula. Attempts to force out the obstruction by doubling the oxygen flow or forcing the saline or cleaning rod through the catheter can result in injury to the trachea 18.A 58-year-old patient with advanced emphysema is admitted with an acute exacerbation of his condition. While breathing 2 L/min of oxygen through a transtracheal oxygen catheter, he has the following arterial blood gas results: pH, 7.38 PaCO2, 57 torr HCO3, 31 mEq/L PaO2, 47 torr SaO2, 80% Based on these findings, what do you think should be done? A.Change the patient to 24% oxygen by an air entrainment mask. B.Initiate bilevel mask ventilation. C.Change the patient to a nonrebreathing

mask with 10 L/min of oxygen. D.Increase the oxygen flow to the current system to 3 L/min. D. The patient's current blood gases show unacceptable hypoxemia with a PaO2 of 47 torr and an SaO2 of 80%. Increasing the patient from 2 to 3 L/min through the transtracheal oxygen catheter should help to correct the situation. The patient should be monitored for a possible increasing carbon dioxide level as well as an improving oxygen level. Changing the patient to 24% oxygen by an air entrainment mask probably will not change his actual inspired oxygen percentage. In addition, no indication that the transtracheal oxygen catheter has failed is reported. The patient's ventilation is stable, with no indication that bilevel mask ventilation is needed. Changing the patient to a nonrebreathing mask with 10 L/min of oxygen is potentially dangerous. This much oxygen may blunt his hypoxic drive to breathe. 19.A comatose patient is intubated and is receiving 35% O2 with aerosol through a Tpiece (Briggs adapter). While watching the patient breathe, you notice that during each inspiration, the mist disappears from the downstream end of the T-piece. What would you recommend to help mange the patient? A.Add 100 mL of aerosol tubing as a reservoir on the T-adapter. B.Change the O2 to 30% and increase the flow. C.Change the O2 to 40% and decrease the flow. D.Tell the patient not to breathe so deeply. A. Adding 100 mL of aerosol tubing as a reservoir will help to maintain the patient's inspired O2 percentage. No clear reason exists for changing the patient's O2 percentage by protocol, and no physician order exists to do so. Changing the flow

probably will not stabilize the patient's O2 percentage without the added reservoir tubing. A comatose patient will not follow instructions to not breathe so deeply. 20.A 65-year-old patient with pulmonary edema is very short of breath and hypoxic. She is ordered to have a nonrebreathing mask with 10 L/min of oxygen going to it. However, she keeps taking off her mask because of anxiety and claustrophobia. When she removes the mask, her pulse oximeter reading drops from 90% to 82%. What should be recommended to help manage the patient? A.Give her a nasal cannula at 10 L/min oxygen. B.Sedate the patient so that she will keep on her nonrebreathing mask. C.Initiate CPAP by mask at 8 cm water and 40% oxygen. D.Begin a high-flow nasal cannula at 10 L/min oxygen. D. A high-flow nasal cannula should be able to provide enough oxygen to the patient to correct her hypoxemia. In addition, she should keep it on because it will not make her feel claustrophobic. A traditional nasal cannula is not run at over 6 L/min of oxygen. It could be dangerous to sedate a hypoxic patient. It is likely that the CPAP mask also will make her feel claustrophobic. 21.An 18-year-old patient with status asthmaticus is being treated in the Emergency Department. The physician has ordered him to receive a 70% helium/30% oxygen mix. What do you recommend as the best way to deliver this? A.Partial rebreather mask with reservoir bag B.High-flow nasal cannula C.Nonrebreather mask with reservoir bag

D.Through a mechanical ventilator C. A nonrebreather mask with reservoir bag is the best way to deliver heliox because it will provide for all of the patient's inspiratory flow needs. A partial-rebreather mask with reservoir bag is not a sealed system. Room air will be entrained, which will reduce the patient's helium and oxygen percentages. A high-flow nasal cannula is designed to provide oxygen, not heliox mixes. There is no indication at this time that mechanical ventilation is needed. In addition, several technical challenges are associated with providing heliox through most mechanical ventilators. 1.Your patient is quite weak and is unable to raise the ball marker on a flow displacementtype incentive spirometer to her set goal. She is becoming discouraged. What would you recommend? A.Have her continue trying. B.Recommend that she be switched to IPPB. C.Change her to a volume displacement type spirometer. D.Discontinue the treatment because it is not effective. C. Switching the patient from a flow displacement to a volume displacement incentive spirometer lets her see the results of her breathing efforts. This should help to motivate her to keep trying. 2.A 16-year-old postoperative appendectomy patient has clear breath sounds and normal vital signs. What would you recommend to prevent atelectasis? A.CPAP at 5 cm H2O B.PEP therapy C.IPPB D.IS D. IS is the most reasonable treatment for

atelectasis at this time. The other three options are more equipment and labor intensive and therefore are more expensive. 3.If pulmonary function results are not available, what initial IS goal should be set? A.The IC measured at the bedside B.The VC measured at the bedside C.Three times the VT measured at the bedside D.Twice the VT measured at the bedside D. An initial IS goal of twice the VT is widely accepted as a starting volume. A bedside IC may be less than that measured under laboratory conditions. 4.It is recommended that IS be performed how often while the patient is awake? A.At least 5 times/hr B.At least 10 times/hr C.At least 20 times/hr D.At least 10 times/day B. It is widely accepted that an IS breath should be performed 10 times/hr while the patient is awake. Less often would be beneficial but not ideal; more often is probably not of any added benefit. 5.Your patient has just performed several excellent IS efforts. She complains of tingling fingers and dizziness. Your response would be to A.Have her continue with additional IS maneuvers. B.Check her fingers and forehead for cyanosis. C.Call her physician to cancel the treatment order. D.Tell her to relax and breathe quietly until she feels normal again. D. Tingling fingers and dizziness are signs of acute hyperventilation. Relaxing and

breathing normally will restore the CO2 level. 6.Your patient has a flow displacement type of IS device. She is attempting but is unable to inhale forcibly through it. What is the most likely problem? A.The inspiratory tube is obstructed. B.The patient is not really trying. C.The flow resistance is set too high. D.The bellows is in the locked-down position. A. A pinched breathing tube is the simplest reason for the obstruction. No special flow resistance features exist on these units. 7.A patient's IS device is a flow displacement type. With good coaching, he can raise a ball with 900 cc/sec of flow. He can keep it elevated for 1.5 sec. What is his IC? A.450 cc B.900 cc C.1350 cc D.1800 cc C. Multiply the volume in 1 second by the number of seconds that the ball is elevated to calculate the inhaled volume. (900 cc/sec 1.5 sec = 1350 cc) 8.Your patient has been using an inspiratory muscletraining device. He is currently on the third largest of six settings and has been breathing comfortably through it 4 days/wk over the past 2 weeks. What would you now recommend that he do? A.Keep breathing through the same inspiratory hole. B.Breathe through the smallest hole. C.Breathe through the largest hole. D.Breathe through the next smallest hole.

D. Because the patient is comfortable breathing at the current resistance level, it is reasonable to have him work a little harder by breathing through the next smallest hole. 9.A patient with advanced COPD is anxious and is feeling short of breath after returning from a medical procedure. What should the respiratory therapist tell the patient to do? A.Turn up the oxygen flow rate to his nasal cannula. B.Inhale through his PFLEX inspiratory muscle trainer. C.Use pursed-lip breathing. D.Exhale through his Flutter valve unit. C. Pursed-lip breathing should help the patient regain control over his breathing and be able to calm down. The patient should not be encouraged or allowed to change his own oxygen flow. This is potentially dangerous. The PFLEX unit is indicated only to help strengthen inspiratory muscles. It will not help to reduce dyspnea. The variable backpressure created by blowing through the Flutter valve unit will only help to mobilize secretions. It will not help to reduce dyspnea. 10.A patient is recovering from a neuromuscular disease and the physician wishes to speed up the process of strengthening his inspiratory muscles. What technique should the respiratory therapist recommend? A.PFLEX inspiratory muscle trainer B.Maximal inspiratory pressure (MIP) C.Maximal expiratory pressure (MEP) D.Trendelenburg positioning A. The PFLEX unit (and other similar units from other manufacturers) is designed specifically to help strengthen the inspiratory muscles. Maximal inspiratory

pressure (MIP) and maximal expiratory pressure (MEP) are bedside spirometry tests to identify a patient's respiratory accessory muscle strength at that time. Neither is intended to exercise the patient's muscles to increase his or her strength over time. Trendelenburg positioning is used in some surgical procedures and in some postural drainage positions. Although the patient may have to work harder to breathe with his or her head down, this position is not used to increase respiratory muscle strength. 1.Ten minutes into a hand-held nebulizer treatment given to deliver albuterol (Proventil), the patient complains of dizziness and tingling fingers. What should be done? A.Advise the patient to breathe in the same pattern. B.Change the medication. C.Tell the patient to breathe more slowly. D.Advise the patient to breathe deeper and faster. C. The patient's signs and symptoms indicate hyperventilation. The easiest way to correct this situation is to have the patient breathe more slowly (and/or less deeply). If the patient continues to breathe in the same pattern, the hyperventilation will continue. Although albuterol may cause tachycardia, it is unlikely to cause dizziness and tingling fingers in the patient. If the patient is advised to breathe deeper and faster, the hyperventilation will get worse. 2.A patient is being given a bronchodilator medication by small-volume nebulizer powered by 5 L/min of O2. While watching the patient breathe, you notice that during each inspiration, the mist disappears from the downstream end of the SVN. What would you recommend?

A.Add 100 mL of aerosol tubing as a reservoir. B.Increase the oxygen flow. C.Decrease the oxygen flow. D.Tell the patient not to breathe so deeply. A. Adding 100 mL of aerosol tubing as a reservoir will help to increase the amount of inhaled medication and maintain the patient's inspired O2 percentage. Increasing the flow of oxygen will make the nebulizer dispense the medication more quickly and waste more of it. Decreasing the oxygen flow may result in the nebulizer not working properly and could decrease the patient's O2 percentage. If the patient were told not to breathe deeply, less medication would be inhaled into the lungs 3.A 4-year-old patient with asthma is about to be discharged and needs to take an inhaled bronchodilator medication at home. What device would you recommend? A.MDI B.DPI C.DPI with holding chamber D.MDI with holding chamber D. An MDI with holding chamber is the only practical way, from among those presented, to deliver this type of medication to a small child. A DPI necessitates an inspiratory flow that is too high for a small child. Holding chambers are not used with a DPI. 4.The physician wants more aerosol inside the mist tent of a 3-year-old child. What would be the best way to do this? A.Cut a hole in the top of the tent. B.Close the hole on the top of the tent. C.Lower the temperature on the refrigeration unit. D.Increase the gas flow to the large-volume

nebulizer. D. Increasing the gas flow on the nebulizer will increase the total amount of water that is nebulized and delivered to the mist tent. Mist tents are supposed to have a hole at the top of the tent canopy to allow the excess gas and aerosol to exit. Only oxygen tents, not mist tents, have the hole closed on the top of the tent canopy. This ensures that the oxygen tent has the highest possible oxygen percentage. Remember, a mist tent does not deliver supplemental oxygen. Lowering the temperature on the refrigeration unit will further cool the infant but will not affect the amount of aerosol delivered into the mist tent. 5.For an aerosolized medication to be deposited primarily in the larger airways, what breathing pattern would you recommend? 1.Inhale a VT. 2.Inhale an IC. 3.Inhale slowly. 4.Inhale at a normal speed. 5.Breathe in a normal pattern. A.2, 3 B.1, 4, 5 C.4, 5 D.2, 4 B. Upper (larger) airway deposition is enhanced by VT breathing at a normal speed and in a normal pattern. A slowly inhaled IC (inspiratory capacity) would increase the change of deposition of medication in smaller airways, not larger airways. 6.A physician calls you to evaluate a 40year-old patient with bronchitis and to make a recommendation for an aerosol delivery system. The patient's breath sounds indicate the presence of large airway secretions.

Despite a good cough effort, the patient has difficulty in raising them. What would you recommend? A.Use a hand-held nebulizer with 3 cc of normal saline every 4 hours. B.Place the patient into a mist tent. C.Start a continuous ultrasonic nebulizer to an aerosol mask. D.Start a cascade-type humidifier to an aerosol mask. C. An ultrasonic nebulizer generates the greatest quantity of aerosol particles from among the listed choices. An aerosol mask is appropriate for an adult. A hand-held nebulizer used every 4 hours does not deliver enough aerosol fast enough to help the patient, as the ultrasonic nebulizer would. A mist tent is not appropriate for an adult. A cascade-type humidifier does not deliver an aerosol that meets the patient's needs. 7.A patient's humidity deficit is going to be the smallest under which of the following conditions? A.Breathing in regular hospital room air at 72 F and 40% relative humidity B.Breathing in outside air at 80 F and 50% relative humidity C.Breathing in 6 L/min of O2 through a nasal cannula running through an unheated bubble humidifier D.Breathing in 40% O2 at 95 F through a cascade-type humidifier to an aerosol mask D. The cascade-type humidifier that is heated to near body temperature will provide almost 100% of the patient's body humidity needs. All the other devices are much cooler than body temperature and provide a lower humidity level. 8.An ultrasonic nebulizer would be

recommended for aerosol therapy for the following reason: A.It delivers a wide variety of aerosol droplets. B.Its aerosol droplets are between 10 and 20 m in diameter. C.It delivers a uniform aerosol droplet of about 3 m in diameter. D.It can be used to nebulize bland aerosols and liquid medications into an aerosol. C. The small, uniform droplet size produced by the ultrasonic unit is a reason for its use. It should not be used to nebulize all medications because vibrations of the ultrasonic unit may break down the drug. 9.Your patient has an endotracheal tube. Which of the following devices would be the least effective in reducing this patient's humidity deficit? A.Wick-type humidifier set at 35 C B.Cascade-type humidifier set at 35 C C.Unheated bubble-type humidifier D.Ultrasonic nebulizer C. An unheated bubble-type humidifier delivers humidity at room temperature or cooler. All of the other units deliver humidity or aerosol at a warmer temperature. 10.An adult patient with chronic bronchitis has a normal temperature. To fully saturate the inhaled air, how much absolute humidity must be provided by the humidifier? A.35 C B.47 mm Hg C.760 mm Hg D.44 mg/L D. The absolute humidity of 44 mg/L is found in the airways of a person at normal body temperature.

11.Your heated humidifier unit has a water reservoir temperature of 40 C. The humidified gas is traveling through largebore tubing to the patient. Which of the following statements are true? 1.Condensation will occur. 2.The gas will warm and expand as it travels to the patient. 3.The gas will remain saturated. 4.The relative humidity will decrease. 5.The relative humidity will increase. A.1, 3 B.4 C.2, 5 D.1, 4 A. Condensation will occur because the saturated air is heated to above room temperature and will cool down as it goes through the tubing. The air will stay saturated with water vapor despite the cooling 12.The pop-off valve is whistling on your patient's bubble humidifier to a 35% O2, air entrainment mask. What could be the problem? A.The reservoir jar is not screwed tightly into the top of the humidifier. B.The air entrainment mask should be set at 28% O2. C.The small-bore tubing is pinched. D.The air entrainment mask should be set at 40% O2. C. Backpressure on the humidifier will make the pop-off valve whistle. Pinched tubing could cause this. The delivered O2 flow does not affect the pop-off valve. If a leak occurs between the reservoir jar and the top of the humidifier, the gas will not leak out of the pop-off valve. 13.A humidity or aerosol system delivering

body temperature gas is used in all the following situations EXCEPT: A.Patient with a tracheostomy B.Twenty-month-old infant with laryngotracheobronchitis C.Patient with chronic obstructive pulmonary disease with thick secretions D.Hypothermic near-drowning victim B. An infant with laryngotracheobronchitis (LTB, or croup) usually is best managed with a cool, bland aerosol. This helps to reduce swelling in the large airways. Body temperature aerosol is delivered to a patient with a tracheostomy or thick (viscous) secretions to minimize the patient's humidity deficit. A hypothermic patient is given body temperature or warmer aerosol to speed up warming. 14.Your patient's small-volume medication nebulizer is not putting out as much aerosol as it was a short time ago. To correct the problem, you would check which of the following? 1.Make sure the fluid level is correct. 2.Make sure the one-way valve is patent. 3.Make sure the jet is patent. 4.Make sure the O2 can flow down the capillary tube. 5.Make sure the fluid can flow up the capillary tube. A.1, 2 B.1, 3, 4 C.2, 4 D.1, 3, 5 D. A nebulizer will fail because of too little water in the reservoir, a clogged jet, or a clogged capillary line to the reservoir. Air entrainment nebulizers do not have one-way valves. O2 flows down a capillary tube in a bubble-type humidifier.

15.Your patient has pneumonia and needs an inhaled antibiotic. What size particle generator would you recommend to treat the problem? A.20 to 50 m B.10 to 20 m C.2 to 5 m D.1 to 3 m D. Particles in the 1- to 3-m range will penetrate to the alveoli. All other sizes will affect the airways. 16.Your ultrasonic nebulizer has a flashing couplant indicator light. You notice that the output has decreased from what it was earlier. The most likely problem is A.Too much water in the solution cup B.Too much water in the couplant chamber C.Not enough water in the couplant chamber D.A loose electrical cable C. Low water level in the couplant chamber will cause the light to flash. No other indicators help with troubleshooting. 17.You notice that water has collected at the low point of the large-bore tubing of your patient's heated aerosol system. The aerosol is puffing out of the end of the tubing. Your reaction should be to A.Add water to the reservoir jar B.Empty water from the reservoir jar C.Empty the water from the large-bore tubing into a wastewater jar D.Empty the water from the large-bore tubing into the reservoir jar so it is not wasted C. Wastewater should be emptied out of the tubing to prevent contamination. The water level in the reservoir jar has nothing to do with water in the tubing. 18.The physician wants a patient with a

tracheostomy to inhale room air that is fully saturated at body temperature. The device that you select must be able to meet the following criteria: 1.Deliver 40% relative humidity 2.Deliver 100% relative humidity 3.Provide 47 mm Hg vapor pressure 4.Provide 44 mm Hg vapor pressure 5.Deliver 44 mg/L absolute humidity A.2, 4 B.1, 3, 5 C.1, 4 D.2, 3, 5 D. The three listed criteria all indicate that the patient's inhaled air is saturated completely with water vapor at body temperature. 19.It is best to coach your patient to breathe in the following pattern for particle deposition in smaller airways and alveoli: 1.Inhale a VT. 2.Inhale rapidly. 3.Inhale an IC. 4.Hold the breath for up to 10 seconds before exhaling. 5.Inhale at a slow speed. A.1, 2 B.3, 4, 5 C.1, 5 D.1, 2, 4 B. The best way for the patient to deposit an aerosol in small airways and alveoli is to slowly inhale an IC with a breath-hold when the lungs are full. A rapidly inhaled VT tends to deposit aerosol in the larger airways 20.When doing patient rounds, you notice that very little aerosol is going from a largevolume nebulizer to a patient's tracheostomy mask. Which of the following could be the

problem? 1.The water level is above the refill line on the nebulizer's reservoir jar. 2.The nebulizer is not screwed tightly into the DISS connector on the flowmeter. 3.The nebulizer jet is obstructed. 4.The water level is below the refill line on the nebulizer's reservoir jar. 5.The capillary tube is obstructed. A.2, 3, 4, 5 B.1, 2 C.3, 4 D.1, 4 A. Any of the following will result in delivery of less or no aerosol: (1) If the nebulizer is not tightly screwed into the DISS connector on the flowmeter, the gas will leak and will not go through the nebulizer; (2) if the nebulizer jet is obstructed, no aerosol will be created; (3) if the water level is below the refill line on the nebulizer's reservoir jar, no water will be drawn up the capillary tube to the jet and baffle; therefore no aerosol will be created; or (4) if the capillary tube is obstructed, the water in the reservoir jar will not be drawn up to the jet and baffle. However, if the water level is above the refill line on the nebulizer's reservoir jar water, water will be drawn up the capillary tube to the jet and baffle. Aerosol will be generated if everything else is functioning normally. 1.A patient with a fluid overload problem has been given a dose of furosemide (Lasix) intravenously. Following rapid diuresis in the patient, you note an arrhythmia that did not exist before the medication was given. What would you recommend? A.Check the patient's potassium level. B.Give more furosemide. C.Defibrillate the patient's heart.

D.Give the patient epinephrine to reverse the allergic reaction to furosemide. A. Lasix is a diuretic and is known to cause patients to urinate large amounts of potassium. The loss of potassium can result in cardiac arrhythmia. Giving more Lasix could worsen the loss of potassium. No indication is given that the patient has a lifethreatening arrhythmia that needs to be defibrillated. No indication exists to give epinephrine. 2.A 2-month-old infant has periods of apnea that result in bradycardia and cyanosis. What medication should be recommended to treat the apnea periods? A.Lidocaine (Xylocaine) B.Neostigmine (Prostigmin) C.Caffeine citrate (Cafcit) D.Isoetharine (Isoetharine HCl) C. Caffeine citrate (Cafcit) is known to be a respiratory center stimulant in infants and will stimulate breathing. This should help to prevent the apnea spells. Xylocaine is a local anesthetic agent. It is used to numb an injury or to stop cardiac arrhythmias, such as premature ventricular contractions (PVCs). Prostigmin is used to reverse the paralyzing effects of nondepolarizing neuromuscular blocking agents such as Pavulon. Isoetharine HCl is a short-acting sympathomimetic bronchodilator. It does not simulate breathing. 3.You are working in the emergency department when an automobile crash victim arrives by ambulance. She is conscious, screaming, and hysterical from the extreme pain of a broken lower leg. What would you recommend for sedation? A.Morphine sulfate (Duramorph) B.Ibuprofen (Advil) C.Succinylcholine chloride (Anectine)

D.Ipratropium bromide (Atrovent) A. Morphine is indicated to relieve severe, acute pain. In addition to controlling the pain, morphine will sedate the patient. Advil is not effective against severe pain. Anectine is a paralyzing medication. Atrovent is a parasympatholytic (anticholinergic) medication. 4.An asthma patient is discontinuing her systemic corticosteroid. She will continue taking her aerosolized bronchodilator. The physician wants to know what you would recommend for an inhaled corticosteroid. A.Naloxone (Narcan) B.Neostigmine bromide (Prostigmin) C.Beclomethasone dipropionate (QVAR) D.Methylprednisolone (Solu-Medrol) C. QVAR is an inhaled corticosteroid that is widely used to treat asthma. Narcan is a reversing agent for morphine and related opium-based drugs. Prostigmin is a reversing agent for nondepolarizing neuromuscular blocking agents. SoluMedrol is a steroid taken by pill or intravenously for systemic effects. 5.Results of your patient's pulmonary function tests show that the peak expiratory flow rate increased the most when she inhaled an aerosolized sympathomimetic drug and an aerosolized parasympatholytic drug. The physician wants to know what you would recommend for this patient. A.Beclomethasone dipropionate (Vanceril) and montelukast (Singulair) B.Ipratropium and albuterol (Combivent) C.Ipratropium bromide (Atrovent) and cromolyn sodium (Intal) D.Salmeterol and fluticasone (Advair) B. Combivent combines a sympathomimetic

(albuterol) agent and a parasympatholytic (ipratropium) agent for effective bronchodilation. Vanceril is an inhaled corticosteroid medication and Singulair is a leukotriene blocker. Atrovent is a parasympatholytic agent and Intal is a mast cell stabilizer. Serevent is a long-duration sympathomimetic agent and Advair is a corticosteroid. 6.Your patient was extubated 30 min ago. She is hoarse and complains of tightness in my throat; inspiratory stridor can be heard. The drug of choice for treating this problem is A.Racemic epinephrine (microNefrin) B.Acetylcysteine (Mucomyst) C.Levalbuterol (Xopenex) D.Isoetharine (Isoetharine HCl) A. MicroNefrin will stimulate the receptors in the mucous membrane of the patient's airway to cause vasoconstriction and reduce edema. Mucomyst is a mucolytic. Xopenex and Isoetharine HCl are sympathomimetic agents with no -receptor effect. 7.Your patient with COPD is coughing very hard to bring up viscous, thick mucus with plugs. What drug of choice would you recommend to treat this problem? A.Dopamine hydrochloride (Intropin) B.Acetylcysteine (Mucomyst) C.Cromolyn sodium (Intal) D.Dornase alfa (Pulmozyme) B. Mucomyst is an effective mucolytic for most patients. Dopamine is a vasoconstrictor used to raise blood pressure. Intal is used to prevent an asthma attack. Hypotonic saline is not known as an effective mucolytic. 8.You receive an order to administer 5 mL of albuterol (Proventil) by handheld

nebulizer. You would proceed to A.Confirm that the order was written and give the treatment B.Have the shift supervisor give the treatment C.Call the physician to check on the medication dose D.Give 0.5 mL of medication because the physician probably intended that dosage C. The physician should be contacted because 5 mL is too large a dose of Proventil. A respiratory therapist should know not to give this large a dose even if the order is written. A respiratory therapist should not alter a written order from a physician or presume to know what a physician might want to do. 9.You are working with a 10-year-old cystic fibrosis patient with a pulmonary infection and thick secretions. What would you recommend to help him cough out the secretions? A.Nebulized 0.9% (normal) saline solution B.Instillation of acetylcysteine (Mucomyst) into his lungs C.Nebulized dornase alfa (Pulmozyme) D.Nebulized acetylcysteine (Mucomyst) C. Pulmozyme is indicated in patients with cystic fibrosis who have pulmonary infection and thick secretions. A normal saline solution is not the most effective option for liquefying secretions. Mucomyst has no effect against purulent secretions with bacterial DNA. In addition, Mucomyst is not currently recommended by aerosol or tracheal instillation in this patient with cystic fibrosis. 10.How much active ingredient would be found in 0.6 mL of 2.25% racemic epinephrine (microNefrin)?

A.26,700 mg B.0.0267 mg C.13.5 g D.13.5 mg D. Review the first two drug dosage calculations shown in Module C to see how to set up this problem. 11.You are administering an aerosolized bronchodilator to your patient. Her pretreatment pulse rate was 85 beats/min. You would stop the treatment if her pulse reached A.90 beats/min B.100 beats/min C.110 beats/min D.120 beats/min C. It is commonly accepted that an aerosolized bronchodilator treatment should be stopped if the patient's pulse rate increases by 20% or more (from 85 to 102 beats/min). 12.Your patient is being given an aerosolized adrenergic (sympathomimetic) drug for the first time. For what possible adverse effects of the medication should you monitor? 1.Bradycardia 2.Tremor 3.Headache 4.Nervousness and irritability 5.Tachycardia A.1, 2 B.3, 4, 5 C.2, 3, 4, 5 D.2, 4 C. Tremor, headache, nervousness and irritability, and tachycardia are all known possible adverse effects.

13.Your patient is being discharged and will receive aerosolized controller-type bronchodilator therapy at home. The best medication for this chronically sick but stable patient is A.Arformoterol (Brovana) B.Metaproterenol (Alupent) C.Levalbuterol (Xopenex) D.Albuterol (Ventolin) A. Brovana is a long-duration medication indicated in stable patients with bronchospasm. The other medications are all sympathomimetic bronchodilators with a shorter duration of action. 14.Your patient has an order for an induced sputum sample to be analyzed for tuberculosis. The best medication for this is A.Dornase alfa (Pulmozyme) B.10% saline solution C.0.9% saline solution D.0.45% saline solution B. Hypertonic (10%) saline is commonly used to induce a sputum sample in patients with tuberculosis. The other saline solutions will not be as effective. Pulmozyme is indicated in patients with cystic fibrosis who have purulent secretions. 15.After finishing an aerosolized dose of acetylcysteine (Mucomyst), a patient has breath sounds that reveal wheezing. These were not present at the start of treatment. What medication should be given before the next Mucomyst treatment to prevent bronchospasm? A.Levalbuterol (Xopenex) B.Sterile water C.Salmeterol (Serevent) D.0.9% saline solution A. Xopenex is a fast-acting

sympathomimetic bronchodilator that should help to prevent a bronchospasm reaction from Mucomyst. Sterile water and normal saline are not bronchodilators and can cause bronchospasm in some asthmatic patients. Nembutal is a barbiturate. 16.You are working the night shift when a 17-year-old patient with status asthmaticus is admitted through the emergency department. She has already been given albuterol (Proventil), Combivent, and an inhaled corticosteroid medication. She has significant tachycardia. The intern on-call asks for your recommendation on what additional medication to give the patient. You would recommend A.Formoterol (Foradil) B.Terbutaline (Brethaire) C.Albuterol (Proventil) D.Theophylline ethylenediamine (aminophylline) D. Aminophylline is recognized as a medication that may be beneficial to a patient with status asthmaticus after inhaled bronchodilators and corticosteroids have been given. If a fast-onset sympathomimetic bronchodilator such as albuterol (in Combivent) has not been effective, it is doubtful if Proventil (more albuterol) or Brethaire would be effective. In addition, the patient is already tachycardic. Foradil is a long-acting sympathomimetic bronchodilator that is not given for a fast effect. 17.A 16-year-old patient has severe and chronic asthma. Her physician wishes to change her medications to prevent her from having asthma attacks. All of the following medications would be helpful EXCEPT A.Zileuton (Zyflo) B.Pirbuterol (Maxair)

C.Cromolyn sodium (Intal) D.Zafirlukast (Accolate) B. Pirbuterol (Maxair) is a fast-onset sympathomimetic bronchodilator. It may be used as a rescue inhaler during an asthma attack but is not used to prevent an asthma attack. Intal, Accolate, and Zyflo are all used as prophylactic agents to prevent an asthma attack. Intal stabilizes mast cells to prevent the release of leukotriene agents. Accolate and Zyflo block the effect of released leukotriene agents. 18.Your patient with COPD is being given a new inhaled adrenergic bronchodilator medication by small volume nebulizer. Within 3 min, he complains of palpitations. His pulse rate was 85 beats/minute before the treatment and is now 125 beats/minute. What should you do? A.Change to a different medication for the next treatment. B.Discontinue the order. C.Stop the treatment and monitor the patient. D.Add more saline to dilute the medication for the next ordered treatment. C. Treatment should be stopped because the patient's heart rate has increased by more than 20%. Monitor the patient's heart rate to find out if it returns to normal; chart the results. Only the physician can order a change in type of medication or medication amount. Only the physician can terminate treatment. Adding more saline to the medication will dilute the mixture but not reduce the amount of medication the patient receives. The same adverse reaction will probably happen again. 19.You have finished giving your 8-year-old patient with asthma an IPPB treatment with 0.5 mL of albuterol (Proventil). You notice

that her heart rate has increased 15% from before the treatment. Her breath sounds are now clear. What would you recommend to the physician for your patient's next treatment? A.Give her 0.5 mL of metaproterenol (Alupent). B.Discontinue her treatments altogether. C.Add more normal saline to the Proventil. D.Decrease the Proventil to 0.3 mL. D. Decreasing the dose of Proventil is reasonable because it may be causing the patient's heart rate to increase. Adding saline will not reduce the total amount of medication that the patient will receive. There is no need to stop the treatment at this point based on the 15% increase in her heart rate. 20.You are called to the recovery room to assist in the care of a patient who returned 2 hr ago from having a bowel resection. The patient is apneic and on a mechanical ventilator. Which medication(s) could be used to wean her from the machine? 1.Flumazenil (Romazicon) 2.Naloxone (Narcan) 3.Dopamine (Intropin) 4.Succinylcholine chloride (Anectine) 5.Diazepam (Valium) A.1, 4 B.5 C.2, 3 D.1, 2 D. Romazicon reverses a barbiturate medication, and Narcan reverses a narcotic medication in the patient. Intropin is used to raise blood pressure. Anectine is a depolarizing neuromuscular blocking agent. Valium is a barbiturate.

21.How much metaproterenol (Alupent) is needed to give the patient 20 mg of the active ingredient? Alupent contains 5% active ingredient. A.1 mL B.0.4 mL C.40 mL D.0.1 mL D. Review the third and fourth drug dosage calculations shown earlier in the chapter to see how to set up this problem. (Module C) 22.An anxious 10-year-old asthma patient is being given a breathing treatment with levalbuterol (Xopenex). His initial heart rate of 110 breaths/minute has now increased to 120 breaths/minute. What should you do? A.Continue the treatment. B.Stop the treatment. C.Change to albuterol and continue the treatment. D.Cut the medication dose in half and continue. A. The patient's treatment can be continued because his heart rate has not increased by at least 20% from the baseline. The therapist cannot change a medication or decrease a medication amount without the physician's approval. 23.The preliminary laboratory results of a sputum sample that was sent from your patient indicate that she has a gram-positive pulmonary infection. Which of the following medications would you recommend? A.Ribavirin (Virazole) B.Penicillin (Ampicillin) C.Tobramycin (TOBI) D.Gentamycin (Garamycin) B. Ampicillin is used against gram-positive bacteria. Virazole is used against RSV.

Garamycin and TOBI are used against gramnegative bacterial pneumonia. 24.An 18-month-old infant is diagnosed with bronchiolitis from RSV. Which medication would you recommend? A.Ribavirin (Virazole) B.Pentamidine isethionate (NebuPent) C.Gentamycin (Garamycin) D.Trimethoprim and sulfamethoxazole (Bactrim) A. Virazole has been approved by the Food and Drug Administration for the treatment of RSV. NebuPent is nebulized and Bactrim is given systemically against P. carinii. Garamycin is used against gram-positive bacteria. 1.A 15-year-old patient with cystic fibrosis has copious amounts of secretions. She cannot tolerate PDT because she gets a headache when tipped head-down. Aerosolized bronchodilators and mucolytic agents are ordered every 4 hr by SVN. What else would you recommend? A.Add incentive spirometry. B.Give the aerosolized medications by IPPB therapy. C.Add PEP therapy. D.Modify the PDT positions so that the head is not lower than the patient's body. C. PEP therapy has been shown to help in mobilizing secretions. In addition, some PEP units can be coupled with an SVN to more efficiently deliver the medication. Incentive spirometry does not help with secretion mobilization or medication delivery. IPPB may help with medication delivery if the patient cannot properly perform the SVN treatment. However, no indication of this problem exists. The benefits of improperly positioning a patient for PDT are questionable. (See Box 10-5 on

page 296.) 2.To get the best patient results, manual percussion should be performed with 1.The hand cupped 2.A tight, fixed-wrist position 3.The elbows relaxed 4.The hand flat 5.The wrist relaxed A.3, 4, 5 B.2, 3, 4 C.2, 4 D.1, 3, 5 D. Manual percussion should be performed with a cupped hand and relaxed wrist and elbow joints. 3.How should manual vibration be performed as part of CPT? 1.On inspiration 2.At a rate of 20 to 30 cycles/sec 3.On expiration 4.At a rate of 3 cycles/sec 5.Throughout the breathing cycle A.2, 4 B.1, 4 C.3, 4 D.4, 5 C. Vibration should be performed only on expiration. Most people cannot vibrate at a faster rate than about 3 cycles/second. 4.When a patient's chart is reviewed, it is important to look for contraindications to CPT. They would include all of the following EXCEPT: A.Increased intracranial pressure in a patient with a recent head injury B.Recent stroke C.Small vital capacity in a bedridden patient D.The patient has just eaten C. There is no reason that a bedridden

patient with a small vital capacity cannot have CPT. Head-down positions place a patient with a recent stroke or known increased intracranial pressure at risk for further brain damage. A patient who has just eaten should not be placed in a head-down position because of the risk of vomiting. (See Box 10-7 on page 297.) 5.A stroke patient has been admitted and is in a coma and unable to care for himself. The physician is concerned that he may develop atelectasis and pneumonia. What would you recommend to help prevent these problems? A.Regular turning from side to side B.PEP therapy C.IPPB D.CPAP A. Regular turning is an easy and inexpensive way for the patient to alter his breathing pattern and to move the VT into different lung segments. This helps to prevent atelectasis. PEP and IPPB may be needed, but only after regular turning has been shown to be ineffective. CPAP is not indicated for the treatment of simple atelectasis from inactivity. 6.You receive an order to perform postural drain-age, percussion, and vibration on a patient. No segments are specified. On reviewing the chest x-ray film, you notice infiltrates in the lower right lung field. You would proceed to treat the following segments: 1.Apical 2.Lateral basal 3.Superior 4.Medial 5.Posterior basal A.2, 5 B.3, 4, 5

C.1, 4 D.2, 3, 5 D. The lateral basal, superior, and posterior basal segments are all located in the right lower lobe where the infiltrates are located. The apical segment is located in the upper lobe, and the medial segment is located in the middle lobe. (Review Figure 10-1 on page 289.) 7.A physician has ordered PEP therapy with albuterol. All of the following are needed to start the treatment EXCEPT: A.Variable orifice resistor B.Pressure manometer C.Bedside spirometer D.Nebulizer with reservoir C. A bedside spirometer is not needed because the patient's exhaled volume does not need to be measured. In addition, a spirometer cannot be connected to the unit for volume measurement. All of the other listed items are needed. The variable orifice resistor is needed to set the level of expiratory resistance. A pressure manometer is needed to determine that the expiratory pressure is kept in the 10- to 20-cm water range. A small volume nebulizer with reservoir is needed to deliver the albuterol. (See Figure 10-17 on page 299.) 8.All of the following are contraindications to percussion and vibration EXCEPT: A.Performing the procedure over the kidneys B.Mobilizing large amounts of secretions C.Performing the procedure over bare skin D.Performing the procedure over or near a surgical site B. Many references indicate that percussion and vibration are beneficial in mobilizing large quantities of secretions. Review Box

10-2 on page 287 for contraindications for percussion and vibration 9.A patient is positioned on her left side with the foot of the bed raised 18 inches. She would be draining which lung segment? A.Anterior basal B.Superior C.Lateral and medial lingular D.Posterior basal A. Review Figure 10-4 on page 290 for the recommended position to drain the anterior basal segment of the right lower lobe. 10.Your patient has been ordered to start PEP therapy. During the initial instruction and patient practice, you notice that the pressure is 25 cm H2O and the patient's I:E ratio is 1 : 5. You would proceed to A.Adjust the PEP device to have the patient exhale through a larger hole B.Have the patient continue but coach him to exhale faster C.Adjust the PEP device to have the patient exhale through a smaller hole D.Add a bronchodilator medication to the PEP device A. When the patient exhales through a larger hole, the pressure should decrease and the expiratory time should shorten. Review Box 10-8 on page 297 for the steps to be followed in the PEP therapy procedure. Having the patient exhale faster or exhale through a smaller hole increases the pressure and expiratory time. A bronchodilator medication has not been ordered for the patient. 11.When you are reviewing a patient's chart, it is important to look for indications for postural drainage. These would include all of the following EXCEPT:

A.A patient with bronchiectasis and retained secretions B.A patient with cystic fibrosis who has retained secretions C.Draining of an empyema D.Removal of an aspirated foreign body C. Because an empyema is a collection of pus in the pleural cavity, it cannot be drained through postural drainage. The other options are all indications for postural drainage. See Box 10-1 on page 286 for the indications. 12.You receive an order to perform postural drainage, percussion, and vibration on a 23year-old female patient. The lateral and medial segments of the right middle lobe are among those that need to be treated. You would proceed to A.Drain, percuss, and vibrate the segments B.Drain and vibrate the segments C.Drain but not percuss or vibrate those segments D.Drain and use a mechanical percussor C. Neither manual nor mechanical percussion or vibration should be performed over female breast tissue. (See Box 10-2 on page 287.) 13.A patient is using the Flutter and coughs productively. Later, the patient tries to use the device but finds that no air will go through it. What should be done? A.Have the patient breathe in harder. B.Check for an obstruction. C.Remove the steel ball to reduce the backpressure. D.Have the patient blow out harder. B. It is likely that the patient coughed secretions into the unit. Check for an obstruction. If one is present, it must be removed. A cotton swab or warm running

water should remove any secretions. The patient blows out (does not breathe in) through the Flutter valve. It is not an incentive spirometer device. Removing the steel ball from the device will prevent it from working as intended. If the patient blows out hard, the obstruction may be blown deeper into the unit. 14.A patient with bilateral pneumonia is positioned for drainage of the lateral and medial segments of her right middle lobe. After 5 min in this position, the patient complains of SOB. The electrocardiogram shows the patient to be having premature ventricular contractions (PVCs). The most likely cause of this is A.Hypoxemia B.A full stomach that is causing vagal stimulation C.Increased intracranial pressure D.Increased venous return to the heart A. Hypoxemia is the only possibility for the problem from among those listed. A full stomach does not cause vagal stimulation that results in PVCs. The patient's symptoms of SOB and PVCs do not correspond with increased intracranial pressure or increased venous return to the heart. 15.You are using a pneumatically powered mechanical percussor on a patient receiving CPT. The unit is powered by an E-cylinder of O2 because piped-in O2 is unavailable. After a few minutes of operation, you notice that the percussor begins to slow down and then stops. What would you do? A.Switch to an electrically powered percussor. B.Make sure the cylinder is completely turned on. C.Check the unit's batteries. D.Check the electrical cord.

A. Switching to an electrically powered percussor is the only workable option from among those provided. A pneumatically powered percussor does not have batteries or an electrical cord. The way that the percussor gradually lost function indicates that the tank did not have any compressed O2 with which to run the unit 16.A mechanical percussor is ordered to assist with secretion clearance in a patient receiving CPT. The patient is positioned to drain the posterior basal segments of both lower lobes. The percussor is activated and applied to the patient's lower back. After 1 minute, the patient complains of skin discomfort. What should the respiratory therapist do? A.Have the patient sit up. B.Apply oxygen and check the pulse oximeter value. C.Increase the speed on the percussor. D.Change to another type of pad on the percussor. D. Try another type of pad on the percussor to determine whether it is more comfortable for the patient. A flat one is probably best for the lower back. The patient is not having a reaction to the head-down position and does not need to sit up. No indication exists that the patient is hypoxic. Increasing the speed on the percussor is likely to increase the skin irritation. 17.A home care cystic fibrosis patient is starting HFCWO therapy. You would instruct the patient to do all of the following EXCEPT: A.Sit upright for the procedure. B.Set the controls to initially deliver the fastest rate and highest pressure. C.Connect the air hoses to the vest. D.Set the controls to initially deliver the

slowest rate and lowest pressure. B. It is unnecessarily risky to set the HFCWO controls at maximal speed and pressure to start a patient's treatment. It could be uncomfortable and the patient could be injured. Start the treatment at the lowest rate and pressure and let the patient increase the rate and pressure as needed. The patient should sit upright for the treatment to be most effective. The pumping unit and vest must be connected with the air hoses. 1.A fully compensatory pause is seen after which type of heartbeat? A.Normal sinus rhythm B.PVC C.Paroxysmal atrial tachycardia D.VT B. See Figure 11-36 on page 331 for a tracing and explanation of a PVC. 2.All the following are acceptable ways to ventilate a patient during CPR EXCEPT: A.Endotracheal tube B.Pneumatic (demand-valve) resuscitator C.Mouth-to-valve resuscitator D.Manual resuscitator B. Current CPR guidelines state that effective ventilation can be achieved by an endotracheal tube, mouth-to-valve resuscitator, or manual resuscitator. A pneumatic (demand-valve) resuscitator is not recommended for use because it is difficult to control the delivered tidal volume and air tends to be forced into the patient's stomach. 3.A patient comes into the Emergency Department appearing ashen gray and complaining of sudden, severe pain beneath his sternum and shortness of breath. He says this began after he exercised

vigorously for 45 minutes. After putting an O2 mask on the patient, what should you do? A.Start ECG monitoring. B.Recommend that he begin a supervised exercise program at the hospital. C.Perform a peak flow test to check on exercise-induced asthma. D.Immediately draw an ABG sample. A. ECG monitoring is justified because the patient's signs and symptoms could indicate a cardiac problem. An exercise program is not indicated in this situation and could be dangerous for the patient. A peak flow test is not indicated now and would not help with the diagnosis of exercise-induced asthma. It is best to wait at least 10 minutes after putting O2 on a patient before drawing an ABG sample to check on the patient's O2 level. 4.The nurse calls you into a patient's room. You notice from the ECG monitor that the patient is in ventricular tachycardia. You cannot find a carotid pulse, and the nurse says that he cannot find a blood pressure. What would you recommend? A.Check the other arm for a blood pressure. B.Defibrillate the patient. C.Intubate the patient and start her on a ventilator. D.Initiate synchronized cardioversion of the patient. B. Defibrillation is indicated if the patient has VT and is without pulse or blood pressure. The patient should then be evaluated for full CPR efforts. The other options would delay effective treatment. 5.Counting from the left, the first and sixth rhythms on the ECG strip shown here represent:

A.Atrial flutter B.Second-degree heart block C.Unifocal PVCs D.Multifocal PVCs

C. See Figure 11-36 on page 331 for a tracing of a PVC and explanation. Unifocal means that all of the PVCs originate from a single area. Multifocal means that PVCs originate from more than one area. 6.A 65-year-old patient has been successfully resuscitated in the Emergency Department after suffering an MI. He is still unstable with frequent PVCs. He needs to be transported to the cardiac care unit for management. Which of the following would be most important for monitoring him during the transportation? A.Pulse oximeter B.Portable capnography unit C.Portable ECG machine with defibrillator D.12-lead ECG unit to record any arrhythmias C. A portable defibrillator must be with the patient in case it is needed. The other items are useful for monitoring but offer no way to treat a life-threatening arrhythmia. 7.You are performing chest compressions during a resuscitation attempt while another therapist is manually ventilating the intubated patient. The nurse and physician are both unable to start an IV line to give medications. What would you recommend? A.Instill the medications down the endotracheal tube. B.Keep trying new sites from which to start

the IV line. C.Nebulize the medications. D.Give the medications by subcutaneous injection. A. Direct instillation into the patient's airways and lungs offers the fastest way to administer the medications when an IV line is not available. 8.You are doing O2 rounds on patients in the coronary care unit. You notice that the patient whose 28% venturi mask you are checking is unresponsive to your questions. Looking up, you see the ECG rhythm strip shown here. What would you recommend as a first reaction? A.Check the calibration on the ECG machine. B.Replace the ECG leads. C.Increase the O2 percentage because the patient is hypoxic. D.Defibrillate the patient.

of normal. This results in reversal of the ECG signal. A loose electrode or shivering would cause different types of artifacts. Miscalibration would not cause inversion of the QRS complex 10.You enter a patient's room to check on her nasal cannula. She is slumped over in her chair and appears cyanotic. Your first reaction would be to: A.Open the airway. B.Determine whether the patient is responsive. C.Begin mouth-to-mouth ventilation. D.Check for a pulse. B. The first step in a suspected CPR situation is to assess the patient. CPR should be performed only when needed. 11.You are working in the Emergency Department when an automobile accident victim is brought in. You suspect that the driver has a cervical spine injury. What is the best way to open the airway? A.Tracheostomy B.Head-tilt/chin-lift maneuver C.Jaw-thrust maneuver D.Nasal intubation C. The jaw-thrust maneuver should be a safe way to open the airway of a patient with a known or suspected cervical spine injury. The patient's head should not be tilted back because of the possibility of spine injury. Nasal intubation and tracheostomy should be performed, if needed, only after the jawthrust maneuver has been tried. 12.To determine breathlessness, it is best to: 1.Feel for air movement with your cheek by the victim's mouth. 2.Feel the chest rise and fall with your hand. 3.Listen for air movement with your ear by

D. Defibrillation should be performed as quickly as possible when a patient is in VF. Figure 11-40 shows another example. All of the other options delay effective treatment. 9.You notice during a diagnostic ECG that the QRS complex is inverted on lead II. What would most likely cause this? A.An electrode is loose. B.The patient is shivering. C.The arm electrodes are reversed. D.The unit is out of calibration. C. Reversing the arm electrodes results in the heart's electrical signal being received by the ECG machine in the opposite direction

the victim's mouth. 4.Look at the victim's chest for a rising and falling movement. 5.Look at the victim's face for nasal flaring. A.1, 2 B.3, 5 C.1, 3, 4 D.4, 5 C. Look, listen, and feel for air movement to determine breathlessness. Feeling with a hand for chest movement is helpful. However, both hands should be used to hyperextend and support the patient's neck and head. 13.The best way to determine pulselessness in a 10-month-old infant is by checking: A.Brachial pulse for 5 to 10 sec B.Carotid pulse for 3 to 5 sec C.Femoral pulse for 3 to 5 sec D.Brachial pulse for 3 to 5 sec A. A 5- to 10-second check of the brachial pulse should be done to determine pulselessness because an infant's pulse is more difficult to check at the carotid or femoral sites. 14.While doing O2 equipment rounds, you come upon a cyanotic patient who is not breathing. As you reposition the patient and hyperextend his neck, you notice that he has open lip ulcers. What would be the best way to ventilate this patient? A.Perform mouth-to-mouth ventilation. B.Use a mouth-to-valve device stored in the room for this purpose. C.Run to the CPR crash cart and get a manual resuscitation bag and mask. D.Wait for the anesthesiologist to intubate the patient's airway, then use a manual resuscitation bag.

B. A mouth-to-valve device allows for quick ventilations without risk to the rescuer from the patient. Mouth-to-mouth ventilation should be avoided if possible in this situation. The other options would unnecessarily delay ventilations. 15.When one rescuer is performing CPR on an unintubated adult, what chest compression:ventilation ratio should be used? A.5 : 1 B.5 : 2 C.15 : 2 D.30 : 2 D. The current BCLS guidelines indicate a compression:ventilation of 30 : 2 for one rescuer. The other three options do not follow current guidelines with an adult. With an unintubated infant or child, two rescuers would use a compression:ventilation ratio of 15 : 2. 16.To ensure that a manual ventilator is ready for use, you would: 1.Make sure that no gas escapes through the outlet port when it is closed off and the bag is squeezed. 2.Squeeze the bag, and make sure that the air/O2 reservoir intake valve closes properly. 3.Squeeze the bag, and make sure the nonrebreathing valve opens properly. 4.Feel for air leaving the outlet port when the bag is squeezed. 5.Squeeze the bag, and make sure that the air/O2 reservoir intake valve opens properly. A.1, 2, 3, 4 B.1, 2, 5 C.4, 5 D.2, 3 A. All are correct except that the air/O2

intake valve should not open when the resuscitation bag is squeezed. This allows the gas to escape rather than be directed to the patient. 17.Blood for an ABG measurement needs to be drawn during a CPR attempt. Which site would you recommend for this? A.Carotid B.Radial C.Brachial D.Femoral D. The femoral site is recommended because it is a large artery that should be relatively easy to hit and is away from the patient's chest during compressions. 18.A mouth-to-valve resuscitation device is being used on an apneic patient. The respiratory therapist delivers a breath, but the patient's chest does not rise. What should be done next? A.Begin chest compressions. B.Request a lateral neck radiograph. C.Check the valve for proper position. D.Perform abdominal thrusts. C. It is easy and quick to check the valve for proper position. Fix the valve if necessary, and attempt to ventilate the patient again. Because the patient's pulse has not yet been checked, there is no indication that chest compressions are needed. Getting a lateral neck radiograph will greatly delay (probably fatally) ventilating the patient. There is not yet an indication that the patient needs abdominal thrusts to clear an airway obstruction. If the patient cannot be ventilated by the fixed mouth-to-valve resuscitation device, check for an obvious obstruction in the mouth or throat. Reposition the head, and attempt to ventilate again. If the patient still cannot be ventilated, then perform abdominal thrusts.

19.After a 10-year-old child has been intubated, what compression:ventilation ratio should the two rescuers use? A.3 : 1 B.5 : 1 C.30 compressions: 2 ventilations per minute D.100 compressions: 8 to 10 ventilations per minute D. Current BCLS guidelines indicate that two rescuers should use 100 compressions: 8 to 10 ventilations per minute with an intubated patient. A 3: 1 ratio is used on a neonatal patient. The other two options do not follow current guidelines for two rescuers with any patient. 1. You are preparing a stainless-steel type laryngoscope handle and blade for an anesthesiologist. The light does not shine. Which of the following would you do to fix the problem? 1. Get a smaller blade to fit the handle. 2. 3. 4. one. Get a larger blade to fit the handle. Tighten the light bulb. Replace the handle with a plastic

5. Check the batteries and replace them if necessary. A. 1, 4 B. 2 C. 3, 5 D. 4 C. Common problems with intubation equipment include a loose or burned-out

light bulb in the laryngoscope blade or depleted batteries in the handle. Replacing the blade with one that is larger or smaller than appropriate would make the intubation procedure more difficult and dangerous. A plastic handle cannot be used with a stainless-steel blade. 2. An oropharyngeal airway would be indicated under which of the following conditions? 1. To maintain an airway before performing a tracheostomy 2. Seizure activity is expected or present 3. To treat an unconscious patient in the supine position with a soft-tissue upper airway obstruction 4. To stabilize the mouth in a patient with a traumatic jaw injury 5. An orally intubated patient is biting the tube A. 3, 5 B. 4, 5 C. 1, 4 D. 2, 3, 5 D. An oropharyngeal airway is indicated to open the airway of an unconscious patient, protect the airway of a patient with seizures, and prevent an oral endotracheal tube from being bitten. A patient with a tracheostomy does not need an oral airway because he or she is not breathing through the mouth. An oropharyngeal airway probably should not be inserted into the mouth of a patient with a traumatic jaw injury because of the risk that further injury could occur.

3. You have just assisted with the intubation of a normotensive adult patient. To minimize the risk of soft-tissue injury to the trachea, what is the highest endotracheal tube cuff pressure that should be employed? A. Less than 15 cm H2O B. Less than 25 cm H2O C. Less than 35 cm H2O D. Less than 45 cm H2O C. A cuff pressure of up to 35 cm H2O should be safe for a patient with a normal blood pressure. A cuff pressure of greater than 35 cm H2O (>25 mm Hg) is likely to place the patient at risk for damage to the mucous membrane of the trachea 4. All of the following may be used to help determine the position of an endotracheal tube: 1. End-tidal carbon dioxide monitoring 2. An esophageal detection device (EDD) 3. Laryngeal palpation during tube insertion 4. Neck and chest radiographs

5. Observation of bilateral chest movement A. 1, 3, 4 B. 2, 4, 5 C. 1, 2, 3, 4

D. All of the above D. All of these may be used, along with listening for bilateral breath sounds, to confirm the location of the endotracheal tube. An end-tidal carbon dioxide monitoring system (capnograph) or disposable carbon dioxide detector such as the Easy Cap may be used to detect exhaled carbon dioxide. If the tube is placed within the esophagus, no carbon dioxide will be found. An EDD can be attached to the endotracheal tube. After the bulb has been squeezed, it inflates with air from the patient's lungs if the tube is within a major airway. Laryngeal palpation during tube insertion allows the tube to be felt as it passes through the larynx. Remember that these three methods differentiate only between airway and esophageal intubation. They do not detect a tube that has been inserted too deeply and has entered a mainstem bronchus (usually the right). A neck and chest radiograph shows the tube entering the larynx and its tip within the trachea (or a bronchus). Observation of bilateral chest movement is valuable because equal movement correlates with tracheal intubation and ventilation of both lungs. 5.A respiratory therapist replaces a patient's tracheostomy tube with another one of the same size and inflates the cuff with 5 mL of air as was done previously. Immediately, the patient has difficulty breathing and no air can be felt coming from the tube. What could be the problem? A. The tip of the tube has been placed into the subcutaneous tissues. B. The patient has closed her epiglottis over the trachea.

C. More air must be added to the cuff to form a seal. D. The tube has accidentally been placed into the esophagus. A. If the tip of the tracheostomy tube has been placed into the subcutaneous tissues the patient will not be able to ventilate at all. The tube must be immediately withdrawn. Because the tracheotomy site is below the larynx, it does not matter if the patient has closed her epiglottis over the trachea. She still can breathe through the tracheostomy tube. It is unlikely that the new cuff requires any more air than the previous one. Even if this were the case, the patient should still be able to breathe through the tube. It should not be possible to place a tracheostomy tube into a patient's esophagus. 6.While assisting with a CPR attempt, the anesthesiologist asks you to get a properly sized endotracheal tube so that the patient's airway can be quickly intubated. The patient is a large, physically fit man. What tube would you obtain? A. A 7.0-mm-ID oral endotracheal tube B. A 10.0-mm-ID nasal endotracheal tube C. An 8.0-mm-ID nasal endotracheal tube D. A 9.0-mm-ID oral endotracheal tube D. A 9.0-mm-ID oral endotracheal tube is appropriate for a large adult male. Also, the oral tube is more appropriate than a nasal tube in an emergency situation. (See Table 12-1 on page 354 for tube sizes.) 7.You have just measured a tracheostomy

tube's cuff pressure with a blood pressure type mercury manometer. The pressure was 17 mm Hg. What would you recommend? A. Leaving the cuff pressure as it is and charting the measured value B. Increasing the cuff pressure to 20 mm Hg C. Rechecking the cuff pressure with a Cufflator device D. Replacing the tube with a larger one A. A cuff pressure of 17 mm Hg should be safe. No need exists to increase the pressure or replace the tube. 8.A 45-year-old female patient is brought into the emergency department from an automobile accident. She has facial trauma, including a broken nose and jaw. Because of heavy bleeding into her mouth, she is having difficulty breathing. Which of the following would you recommend to ensure a safe, effective airway? A. Place an oral airway. B. Place a tracheostomy tube. C. Place a nasopharyngeal airway. D. Place a nasal endotracheal tube. B. A tracheostomy tube is indicated in a patient with trauma to the nose and mouth and an upper airway obstruction. All of the other airway devices would pass through the upper airway. 9.While working in the neonatal intensive care unit, you are called to assist in the care

of a newborn. The neonatologist asks you to get the proper endotracheal tube size for a premature newborn. What size tube would you obtain? A. 1.5-mm ID B. 2.5-mm ID C. 3.5-mm ID D. 4.0-mm ID B. A 2.5-mm-ID tube would be most appropriate. (See Table 12-1 on page 354 for tube sizes.) 10.A 59-kg (130-lb) woman must be intubated to initiate mechanical ventilation. What size tube should be used? A. 6.0-mm ID B. 6.5-mm ID C. 7.5-mm ID D. 9.0-mm ID C. An adult woman normally has a 7.5 (or 8.0) mm ID endotracheal tube. Review Table 12-1 for the recommended sizes of endotracheal and tracheostomy tubes for patients of all sizes. 11.You are going to assist in the ambulance transport of a 25-year-old patient. The patient has an oral endotracheal tube, and you are going to manually ventilate his lungs during the trip. Which of the following would you choose to help you be sure that the endotracheal tube stays properly placed within the trachea? A. Pulse oximeter B. Capnograph

C. Disposable exhaled-CO2 detector D. Electrocardiogram C. A disposable exhaled-CO2 detector would be easy to use and would give an immediate indication if the tube were removed from the trachea. A capnography device is expensive and is not designed for easy use in a transport situation. Pulse oximetry will give information on O2 saturation, not on CO2 removal from the lungs. An ECG will not give immediate feedback on the patient's condition related to the tube. If the patient was accidentally extubated, both the pulse oximeter and the ECG devices would eventually give information indicating that the patient is in trouble. However, this information is not specific to the patient who has been extubated. 12. All of the following should be monitored after a patient returns from having a tracheostomy tube placed EXCEPT A. Cuff pressure B. Bowel sounds C. Bilateral breath sounds 1. D. Excessive bleeding 2. B. Bowel sounds should not be affected by the placement of a tracheostomy tube. It is highly unlikely that the tracheostomy tube would be accidentally placed into the esophagus, which can happen during the placement of an endotracheal tube. All other items should be monitored. 13. Auscultation of a recently intubated patient in respiratory failure reveals absent breath sounds on the left side of the chest. The most likely cause of this finding is 3. 4. 5.

A. Placement of the endotracheal tube into the right mainstem bronchus B. Placement of the endotracheal tube into the left mainstem bronchus C. Placement of the endotracheal tube into the esophagus D. A pneumothorax on the right side A. Placement of the endotracheal tube into the right mainstem bronchus would result in the absence of breath sounds over the left lung. Placement of the tube into the left mainstem bronchus would result in the absence of breath sounds over the right lung. Placement of the tube into the esophagus would result in the absence of breath sounds over both lungs. A right pneumothorax could result in absent breath sounds over the right lung, not over the left lung. 14. While working the night shift, you are called to intubate an apneic patient. Which of the following would you need for an emergency oral intubation? Laryngoscope handle Stylet Proper laryngoscope blade 10-mL syringe Magill forceps A. 1, 2 B. 1, 2, 5 C. 1, 2, 3, 4

D. 2, 4, 5 C. Everything listed except the Magill forceps would be needed. These forceps are only used during a nasal intubation procedure. (Review Box 12-4 on page 367.) 15.You are assisting with the extubation of an adult patient. At what point in the procedure should the tube be removed? A. At the end of a peak inspiratory effort B. At the end of a normal exhalation C. At the start of a peak inspiratory effort D. During a forced vital capacity effort A. A patient should be extubated when the lungs are full so that a full coughing effort can follow and clear any secretions in the airways. All of the other options would result in less volume in the patient's lungs. 16.A patient with a tracheostomy has just returned from a series of x-ray procedures. Suddenly, she develops respiratory distress and cannot breathe. Your attempt to pass a suction catheter through the tracheostomy tube does not work. You should proceed to A. Attempt to pass a smaller suction catheter B. Remove the tracheostomy tube C. Attempt to ventilate the patient's lungs with a manual resuscitator D. Insert an endotracheal tube

B. The tracheostomy tube should be removed quickly if there is evidence that the tube is blocked. In addition, the tracheotomy tube should be replaced with a new one so that a secure airway is maintained. 17. After a successful CPR attempt, a patient with an oral endotracheal tube is placed on a mechanical ventilator in the intensive care unit. The respiratory therapist notices that the exhaled CO2 monitor is appropriately changing color with each breath cycle. The patient's breath sounds are present on the right side but diminished on the left side. What is the most likely cause of this situation? A. Left-sided pneumothorax B. Right bronchial intubation C. Malfunctioning exhaled CO2 monitor D. Delivered tidal volume is too small B. Right bronchial intubation is indicated by the presence of the patient's breath sounds on the right side but diminished on the left side. There is no direct evidence of a leftsided pneumothorax. The exhaled CO2 monitor is functioning properly because it is supposed to change color (from dark purple to yellow when exposed to exhaled carbon dioxide) during the breathing cycle. Even a small tidal volume should deliver equal air to both lungs and result in equal breath sounds over both lungs.

18. Your patient is an 18-year-old woman who was found unconscious from a drug overdose. She has severe atelectasis of the left lung caused by lying on her left side

for 2 days. Her right lung is normal. She is going to require mechanical ventilation to open the atelectatic areas. What endotracheal tube would you suggest should be used to properly treat the abnormal lung? A. Double-lumen B. Standard C. Fenestrated tracheostomy D. Wire-reinforced A. A double-lumen tube is indicated because she can receive independent lung ventilation through it. This mode of ventilation would allow her lung with atelectasis to be ventilated differently than her normal lung. None of the other tubes offer this option. 19. Your patient has epilepsy and has been having unpredictable seizure activity. What oral endotracheal tube would you suggest should be used to provide a secure airway? A. Double-lumen B. Preformed C. Wire-reinforced D. Guidable C. A wire-reinforced (armored) tube would prevent her from biting and collapsing the tube during a seizure. None of the other tubes offer this security. 20. A conscious patient is recovering from Guillain-Barr syndrome and is able to breathe spontaneously off of the mechanical ventilator for several hours. She currently has a standard 7.5-mm-ID tracheostomy tube. To help her weaning process but to enable her to be ventilated at night, what should be done?

A. Remove the tracheostomy tube when she is off of the ventilator. B. Substitute a speaking-type tracheostomy tube. C. Replace the current tracheostomy tube with one that is 6.0-mm ID. D. Substitute a fenestrated tracheostomy tube. D. Substituting a fenestrated tracheostomy tube for the standard tube allows her to breathe spontaneously through the upper airway when the inner cannula is removed. This allows her to talk, which can have a very positive emotional impact on the patient. It is probably going too far to remove the tracheostomy tube when she is off of the ventilator. This necessitates removing the tube, covering the stoma, and reinserting the tube later in the day. This can lead to damage to the tracheal tissue. In addition, if the patient's condition suddenly deteriorates while the tracheostomy tube is removed, no secure airway is available. Although a speaking-type tracheostomy tube allows her to speak while on the ventilator, it does not enable her to breathe through her upper airway when she is off of the ventilator as a fenestrated tube allows. Replacing the current 7.5-mm-ID tracheostomy tube with one that is 6.0-mm ID greatly increases the patient's work of breathing. This can fatigue the patient and delay her recovery. 21. A semiconscious patient with many tracheal secretions will need frequent nasotracheal suctioning. What can be done to minimize trauma from the procedure? A. Insert a tracheostomy button

with a speaking valve. B. Insert a nasopharyngeal airway. C. Sedate the patient and insert an oropharyngeal airway. D. Suction through a fenestrated tracheostomy tube. B. A nasopharyngeal airway can be inserted to protect the nasal passage from damage by the suction catheter. An oropharyngeal airway should only be used in unconscious patients and does not protect the nasal passage. There was no mention of the patient having a tracheostomy. 22. Your patient who has a tracheostomy button with attached speaking valve is complaining that it is difficult to breathe. You find that a 12Fr suction catheter cannot be passed through the button. What should be done? A. Place a transtracheal oxygen catheter through the tracheostomy button. B. Force a larger suction catheter through the button. C. Remove the button and orally intubate the patient. D. Remove the speaking valve and assess the patient. D. It is best to remove the speaking valve to determine if that is the cause of the dyspnea. If the patient can now breathe comfortably, attach a new speaking valve to the button. If the patient still cannot breathe comfortably, remove the tracheostomy button and reassess the patient. 23. During a surgical procedure, the

anesthesiologist wishes to protect the patient's airway and provide mechanical ventilation, but does not want to place an endotracheal tube. What airway should be used? A. Combitube B. Laryngeal mask airway C. Oropharyngeal airway D. Nasopharyngeal airway in each nostril B. A laryngeal mask airway is commonly used in the operating room to provide a secure airway without an endotracheal tube. The Combitube is an emergency airway and is not employed in the operating room. Oropharyngeal and nasopharyngeal airways do not provide a secure airway. There is no indication for two nasopharyngeal airways to be used at once. 24.You are called to the delivery room to assist in the care of a newborn diagnosed with macroglossia. What should a respiratory therapist be prepared to do in this situation? A. Maintain a patent upper airway. B. Instill surfactant. C. Initiate mechanical ventilation. D. Provide supplemental oxygen. A. A newborn with macroglossia has a very enlarged tongue and is at risk of upper airway obstruction. The immediate situation may require a respiratory therapist to hold the jaw forward or insert an oropharyngeal or nasopharyngeal airway to maintain a

patent airway. Surfactant, mechanical ventilation, and supplemental oxygen are not needed because newborns with macroglossia have normal lung function. 25.After assisting in an emergency oral intubation procedure of an adult patient, you assess his breath sounds and find them absent in the left lung area. The tube depth marking shows it is at the 29-cm mark at the patient's front teeth. What do you recommend be done? A. Check the chest radiograph results for the tube's placement. B. Pull the tube back to the 24cm mark. C. Push the tube down to the 32cm mark. D. Listen for inspiratory sounds over the stomach area. B. Absent breath sounds over the left lung and a tube depth mark of 29 cm both indicate that the endotracheal tube is inserted too deep. It has entered the right mainstem bronchus. The cuff should be deflated, the tube pulled back to the 24-cm mark at the front teeth, and the cuff reinflated. Check again for bilateral breath sounds to confirm that the tip of the tube is within the trachea. It is not necessary to check the chest radiograph to know that the tube is in the right mainstem bronchus. Pushing the tube deeper will only make it go further into the right mainstem bronchus. It is generally recommended that listening over the epigastric area be done after listening over the lung areas for breath sounds. While there is nothing wrong with listening over the stomach area, there is enough clinical evidence given to confirm the tip of the tube is in the right mainstem bronchus.

26.An unconscious 17-year-old patient has arrived in the emergency department. She was involved in an automobile accident, has a neck injury, and is wearing a neck brace. If she were to show signs of an upper airway obstruction, all of the following could be easily used to maintain the airway EXCEPT A. Oral endotracheal tube B. Nasopharyngeal airway C. Laryngeal mask airway D. Oropharyngeal airway A. It could be challenging to place an oral endotracheal tube because the patient has a neck brace. Because of the neck injury, her head cannot be hyperextended. An anesthesiologist would need to be called to perform the intubation. A nasopharyngeal airway, oropharyngeal airway, or LMA can be easily inserted into the patient without the need to hyperextend her neck. 1. All of the following statements about the use of a Lukens trap are true EXCEPT: A. A vacuum source is needed. B. All connections must be tight for it to work properly. C. Either a suction catheter or a bronchoscope is also needed. D. It is used to collect a sputum sample from a patient with a strong, productive cough. D. Lukens traps are used only to get a sputum sample from a patient who cannot cough productively. 2.Your patient is being mechanically ventilated with 60% oxygen and 8 cm H2O

of PEEP. Twice her SpO2 and blood pressure have decreased when she was removed from the ventilator for suctioning. What should be recommended to prevent this from happening again? A. Switch to a smaller suction catheter B. Switch to a larger suction catheter C. Increase the PEEP to 10 cm H2O before and after suctioning D. Use a closed-system suction catheter D. A closed-system suction catheter allows the patient to be ventilated and keep the PEEP level while suctioning is performed. This should help to prevent hypoxemia. Changing to a smaller or larger diameter open-airway suction catheter will not significantly improve the patient's situation. When the patient is taken off of the ventilator and suctioned, she will become hypoxemic. Additional PEEP will not prevent hypoxemia when the patient is taken off the ventilator. 3.The proper-size suction catheter should be no larger than what fraction of the endotracheal tube's ID? A. B. C. D. B. It is commonly accepted that a suction catheter's diameter should not be greater than one-half the ID of the endotracheal tube. This ensures that the patient has room to breathe around the catheter.

4.Removing the tracheal secretions from your adult patient is difficult when you use 60 mm Hg of vacuum pressure. What should you do? A. Suction for 20 seconds B. Suction more frequently C. Increase the vacuum pressure to 80 mm Hg D. Change from the central vacuum system to a portable one C. Increasing the vacuum pressure from 60 to 80 mm Hg will result in the secretions being removed more quickly. The whole suctioning procedure usually is limited to 15 seconds. Suctioning for 20 seconds will remove more secretions but is also likely to cause hypoxemia. Suctioning more frequently will not prove effective if the suctioning level is too low at 60 mm Hg. A hospital's central vacuum system is more powerful than a portable one and will suction more effectively. 5.If your patient has a room air arterial O2 pressure of 65 mm Hg, the most important step to take to prevent hypoxemia during suctioning is to A. Give the patient 100% O2 before and after the procedure B. Use a large catheter to remove the secretions quickly C. Hyperextend the patient's neck and head D. Use a small catheter so that the patient can breathe around it A. All patients should be preoxygenated before suctioning and given added O2 after

suctioning to quickly restore the presuctioning O2 level. O2 at 100% should be given unless a reason exists to give less. 6. The best positions in which to place a patient before nasotracheal suctioning are 1. 2. Supine Trendelenburg's position

turned on to measure the vacuum level A. The full (maximum) vacuum level is too great to be safely applied to a patient's airway. 8.Your 40-year-old patient has pneumonia in her left lower lobe with a large amount of secretions. What would you recommend to be able to suction her better? A. Use the largest diameter suction catheter available B. Use a suction catheter with a Coud tip C. Use the longest suction catheter available D. Suction for a longer period B. A suction catheter with a Coud tip may help guide the catheter either to the left or to the right mainstem bronchus. In adults, a straight catheter has a tendency to go down the right mainstem bronchus because it comes off of the trachea at the carina at a more acute angle than the does the left mainstem bronchus. (In newborns, no significant difference exists in the angles of the mainstem bronchi from the trachea.) As stated in answer 3, the diameter of the catheter should be no more than one-half the ID of the endotracheal tube. Any adult-size suction catheter is long enough to suction adequately. Increasing the time of suctioning puts the patient at risk for hypoxemia. 9. You notice that a Yankauer suction catheter you are using is cracked. The best thing to do is A. Continue to use it B. Tape over the crack

3. Sniff position (neck and head hyperextended) 4. Semi-Fowler position A. 1, 4 B. 1, 3 C. 2, 3 D. 3, 4 D. Hyperextending the patient's head and neck (sniff position) helps open the airway so that the catheter can be inserted more easily into the trachea. Placing the patient in the semi-Fowler position helps the individual to take a deeper breath as needed. 7.Making sure a central vacuum system is working properly includes all of the following steps EXCEPT: A. Setting the vacuum control to FULL B. Screwing a 500-mL collection bottle tightly onto the vacuum connector C. Attaching 3 feet of vacuum tubing to the tubing connector on the collection jar D. Pinching closed the vacuum tubing when the vacuum is

C. Put lubricating jelly in the crack to seal it D. Replace the catheter D. The best course always is to replace a broken or defective piece of equipment 10. You are preparing to suction a patient for a mucus sample when you notice that the vacuum is not reaching the end of the catheter. All of the following are possible causes of this problem EXCEPT: A. The vacuum is not turned on to the proper level. B. The vacuum tubing, specimen collector, and catheter system are connected so that they are airtight. C. The catheter is plugged with foreign matter. D. The specimen jar is not screwed tightly into the special lid. B. If the equipment is properly assembled and does not have an air leak, the vacuum should reach the end of the catheter. The other three examples would result in failure of the vacuum to reach the end of the catheter. 11. Your patient is receiving mechanical ventilation through an 8-mm oral endotracheal tube. Over the course of the shift, the patient is seen to have more tracheal secretions. What is the best course of action? A. Suction more often B. Suction for longer periods

C. Change to a closed-airway suction catheter D. Administer nebulized atropine A. More frequent suctioning is needed if the patient has more secretions. Suctioning for longer periods puts the patient at risk for hypoxemia. No indication exists (e.g., hypoxemia or therapeutic PEEP) that a closed-airway suction catheter is needed or would be more effective than a standard catheter. Nebulized atropine in a large enough dose reduces secretion production. However, a dose large enough to do this also is likely to cause tachycardia. Atropine is rarely given to control secretions other than in the operating room. A physician's order is needed to give this medication. 12.You are suctioning your patient when the vacuum is lost. You should do all of the following EXCEPT: A. Make sure the vacuum system is working B. Make sure a tight connection exists between the suction catheter and the vacuum tubing C. Check the catheter to make sure it is not obstructed D. Get a larger suction catheter D. A larger suction catheter would not make any difference in restoring vacuum pressure. If the suctioning system is working properly, vacuum should be felt at the end of a catheter of any size. Problems with the other listed options could result in loss of vacuum. 13. A ventilator-dependent patient required a vacuum pressure of 120 mm Hg to remove her thick secretions. After

treatment with a mucolytic drug, her secretions are much easier to remove. What would you recommend? A. Reduce the vacuum pressure to 100 mm Hg and monitor the ease of secretion removal B. Maintain the present vacuum level and suction less often C. Increase the vacuum pressure to 140 mm Hg and suction less often D. Reduce the vacuum pressure to 60 mm Hg and suction more often A. The most reasonable course is to reduce the vacuum level slightly and assess how easily the secretions can be removed. Suctioning frequency should not be reduced unless fewer secretions are produced. The vacuum level need not be increased. 14.A conscious patient requires nasotracheal suctioning. During the procedure, the patient's blood pressure decreases to 100/60 mm Hg, and the heart rate decreases from 110 to 60 beats/min. What should be done? A. Change to a catheter with a larger diameter B. Shorten the suctioning time C. Insert an oropharyngeal airway before suctioning D. Squirt 5 mL of saline down the suction catheter into the patient's trachea B. Because the patient is having an adverse reaction to the suctioning procedure, the suctioning time should be shortened. Changing to a catheter with a larger

diameter increases the risk of trauma to the mucous membrane of the nasal passage. Inserting an oropharyngeal airway before suctioning may stimulate the gag reflex in a conscious patient. In addition, the airway may block the suction catheter, preventing it from passing through the patient's oropharynx to the larynx and trachea. Squirting 5 mL of saline down the suction catheter into the patient's trachea will probably trigger coughing. This could worsen the patient's distress with the whole procedure. 15.The respiratory therapist is called to set up a suctioning system for a new patient in the intensive care unit. To measure the vacuum pressure, what should be done? A. Check the manometer while occluding the catheter tip B. Set the vacuum control to maximum. C. Close the thumb control valve on the catheter D. Check the manometer while pinching off the connecting tubing D. To seal the system and determine the set vacuum pressure, check the manometer while pinching off the connecting tubing. The system is not sealed off if the therapist just occludes the catheter tip; the thumb control valve is still open. Setting the vacuum control to maximum without sealing off the system allows air to leak, and the pressure cannot be measured. The system is not sealed off if the therapist just closes the thumb control valve on the catheter; the catheter tip opening is still open. 16.An intubated patient has pneumonia, and a sputum sample must be sent to the

laboratory for culture and sensitivity testing. What is the most appropriate way to obtain a sample? A. Place a Lukens trap between the suction catheter and the vacuum tubing B. Suction the oropharynx with a sterile Yankauer suction catheter C. Place a Lukens trap between the vacuum tubing and the collection bottle. D. Suction the patient and place the catheter inside the Lukens trap A. The only way to obtain an uncontaminated sputum sample is to place a Lukens trap between the suction catheter and the vacuum tubing. This way, the patient's secretions are collected in the Lukens trap after they have passed through the sterile suction catheter. Suctioning the patient's oropharynx with a sterile Yankauer suction catheter provides an oral sample, not a tracheal sample. The patient's mouth probably contains microorganisms besides those causing the pneumonia. Placing a Lukens trap between the vacuum tubing and the collection bottle would provide a contaminated sample, because unlike a suction catheter, the vacuum tubing is not sterile. The Lukens trap is designed to hold secretions and prevent them from becoming contaminated. It is not designed to hold a suction catheter. 1.Your patient complains of difficulty in starting the IPPB treatment. You would adjust which of the following controls? A. Pressure

B. Flow C. Sensitivity D. Terminal flow C. The sensitivity control on all IPPB units determines how much effort (negative pressure) the patient has to generate to trigger a breath. Pressure, flow, and terminal flow will adjust the functioning of the IPPB unit after the breath is started. 2.Your patient is having difficulty keeping a tight seal around the mouthpiece. He complains that the breath is too long and takes out the mouthpiece. To help cycle off the PR-2, you would adjust which of the following? A. Pressure B. Flow C. Terminal flow D. Expiratory retard C. Terminal flow on a Bennett PR-2 unit is adjusted to attain additional flow at the end of an inspiratory effort. This added flow compensates for a small leak and cycles the unit to exhalation. Pressure, flow, and expiratory retard will adjust the functioning of the IPPB unit after the breath is started. 3.Your patient is progressing to pulmonary edema. She has crackles in both lung fields, has cyanotic lips and nail beds, and is coughing up pink, frothy sputum. What O2 percentage would you recommend for her IPPB treatment? A. 21% B. 40%

C. 80% D. 100% D. Pure O2 is most clearly indicated in a patient with pulmonary edema and signs of hypoxemia. Room air (21% O2) would not be very helpful. Intermediate levels of supplemental O2 (40% or 80%) would be helpful but not as effective as pure O2. 4.All of the following indicate the need for IPPB EXCEPT A. Delivery of medications to a patient who cannot coordinate the use of an MDI or a handheld nebulizer B. Treatment of a comatose patient with atelectasis C. The need for an IS substitute in a patient with an IC that is 30% of predicted D. Treatment of a cooperative patient with atelectasis D. A cooperative patient with atelectasis should first be treated with a less expensive method such as IS. IPPB would be indicated in the other types of patients. 5.You are ordered to give an IPPB treatment to a comatose patient who has lip ulcers. What patient-machine connection would you use? A. Mouthpiece B. Face mask C. Bennett seal with mouthpiece D. Endotracheal tube adapter for intubation

B. A face mask would allow a treatment to be given without injury to the lip ulcers. A mouthpiece cannot be held by a comatose patient. A Bennett seal would injure the lip ulcers. Intubation is unnecessarily invasive and risky. 6. The sensitivity control should be set at what level at the start of IPPB treatment? A. 0 cm H2O B. -1 cm H2O C. -3 cm H2O D. -5 cm H2O B. A small negative pressure of 1 cm H2O would not make the patient work harder than necessary to turn on the unit. A pressure of 0 cm H2O would result in the self-cycling of the unit when ambient (room) barometric pressure is reached. 7.While coaching an active IPPB treatment, you notice that the needle on the pressure manometer bounces around as the pressure increases. To better adjust treatment to the patient's needs, you would do which of the following? A. Increase the flow. B. Decrease the flow. C. Increase the peak pressure. D. Decrease the expiratory retard. A. The needle on the pressure manometer bounces when the patient's inspiratory flow is greater than what is leaving the machine. Turn up the inspiratory flow from the unit for a smoother inspiratory effort by the patient. Decreasing flow would make the

patient's situation worse. Increasing peak pressure will increase tidal volume only. Decreasing expiratory retard would alter the expiratory flow. 8.You are giving an IPPB treatment on a Bird Mark 7 unit. To give the patient 100% O2, you push in the air-mix control knob. What effect does this adjustment have on the flow rate to the patient? A. Decreases the flow of gas B. Increases the flow of gas

There is no indication that the patient is hypoxic and needs 100% oxygen. 10.While giving an IPPB treatment, a hissing sound is heard and the patient complains that the inspiratory time is too long. What is the most likely problem? A. The nebulizer hose is attached to the exhalation valve. B. The nebulizer medication jar is loose. C. The bacteria filter is missing.

C. No effect D. Increases the sensitivity A. When the air-mix knob on a Bird unit is pushed in, only pure source gas (100% O2) is given to the patient. Because no room air is entrained with the source gas, the overall total gas flow is decreased. 9.In the emergency department, you are giving an IPPB treatment with albuterol (Ventolin) to an asthmatic patient. During a break in the treatment, the patient complains that his lungs feel too full and he does not feel like all the IPPB volume is getting out. What would you recommend? A. Increase the flow. B. Add expiratory retard. C. Increase the system pressure. D. Change to 100% oxygen. B. Adding expiratory retard adds some back-pressure to the patient's airways and allows for a more complete exhalation. Increasing inspiratory flow increases turbulence, and increasing system pressure increases tidal volume. Neither of these will help the patient's problem of air trapping. D. The inspiratory and expiratory hoses are reversed. B. A loose nebulizer medication jar would result in a hissing sound from the leaking air and a prolonged inspiratory time. If any of the hoses are misconnected, the IPPB machine will fail to function properly but the described problems will not be found. A missing bacteria filter does not cause any hissing sound or prolong the inspiratory time. 11.A patient who was initially anxious about taking an IPPB treatment and needing a fast breath is now breathing in a more relaxed manner. What adjustment should be made to allow for a longer inspiratory time? A. Decrease the peak pressure. B. Increase the sensitivity. C. Set the air-mix knob to allow room air to be entrained. D. Decrease the flow. D. Decreasing the inspiratory flow would result in a longer inspiratory time. This is recommended to better distribute any inhaled medication. Decreasing peak

pressure results in a smaller tidal volume and a shorter inspiratory time. Sensitivity has no effect on inspiratory time. If room air is entrained, the total flow will increase and inspiratory time will be shorter. 12.A patient you are evaluating has a spontaneous VT of 400 mL and has been diagnosed with atelectasis after bowel surgery. IPPB has been ordered. What would you recommend as the minimal volume goal? A. 300 mL B. 400 mL C. 500 mL D. 800 mL C. The 1993 Clinical Practice Guideline on IPPB recommends that an IPPB volume be at least 25% greater than a patient's spontaneous volume. 13.After 10 min of being given an IPPB treatment, the patient complains of a sharp chest pain. After a few more deep breaths, he says he is short of breath. You notice that his breath sounds are now diminished on the left side. What would you recommend? A. Continue the treatment for the next 5 min to finish the ordered time. B. Decrease the peak pressure and complete the ordered treatment. C. Stop the treatment and notify the physician of the patient's complaints. D. Monitor the patient closely for the duration of the treatment, and notify the nurse of the

patient's complaints. C. The patient's signs and symptoms indicate a pneumothorax. Treatment should be stopped, the patient evaluated, and the physician notified.

1.After instructing a home care patient on the use of her small volume nebulizer, you want to be sure that she understands how to fill and clean it. How could you confirm this? 1. Have the patient demonstrate use of the nebulizer to you. 2. Have the patient's boyfriend demonstrate use of the nebulizer to you. 3. Have the patient answer your questions about the nebulizer. 4. Have the patient tell you how the equipment is assembled and cleaned. A. 1, 3, 4 B. 1, 2 C. 3, 4 D. 2, 3, 4 A. The best way to be sure that the patient understands the purpose of the equipment is to ask her questions about it and have her describe its function. Any misunderstandings then can be clarified. In addition, the patient should show that she has the necessary dexterity to operate the equipment. 2.Your patient with chronic obstructive pulmonary disease (COPD) is starting a pulmonary rehabilitation program and taking her first 6-minute walk test. If she should

desaturate, at what pulse oximeter reading would supplemental oxygen be indicated? A. <96% B. <90% C. <88% D. <84% C. Current guidelines recommend that supplemental oxygen be used during exercise when a patient's SpO2 value drops <88%. Values between 96% and 90% are within the normal range. A value of <84% shows serious hypoxemia. The patient should not be allowed to become this hypoxemic if possible. 3.A home care patient tells you that she cannot feel any O2 coming to her nasal cannula from the O2 concentrator. You would do all of the following EXCEPT A. Refill the humidifier bottle with sterile water B. Place the cannula prongs under water to see if there is any bubbling of gas C. Tighten all the equipment connections D. Switch the patient to her tank of O2 A. The level of water in the humidifier does not affect the ability of O2 to flow through the O2 delivery system. Tightening of all connections and placement of the cannula prongs under water reveal whether gas is flowing through the prongs. If no gas is flowing out from the O2 concentrator, the nasal cannula should be switched over to the O2 tank. (See the Entry Level Exam Hint on page 527.)

4.On surveying a patient's home, you notice that four steps lead to the front door. The patient's bedroom is upstairs, and his wife smokes about one pack of cigarettes a day. You would recommend all the following EXCEPT A. The patient's wife should stop smoking B. The patient should use a cane when climbing stairs C. The patient's bedroom should be moved downstairs D. A ramp should be added to facilitate entering through the front door B. A cane should not be used without a documented need. The patient's life situation can improve if his spouse stops smoking, he minimizes climbing of stairs, and a front door ramp is added. 5.When first visiting a home care patient's house, the therapist should evaluate all of the following to attempt to eliminate risks associated with the patient's performance of daily activities: 1. Bathroom facilities, including an assessment of ease of getting into and out of the shower or bathtub 2. Television and remote control for operation 3. Kitchen facilities, including all cooking and eating utensils placed within arm's reach 4. Properly cleaned HEPA filtration system A. 1

supplemental O2 needed B. 1, 2, 3, 4 C. 3, 4 D. 1, 2 D. Many injuries occur in the bathroom and kitchen when weak patients fall while climbing into showers or tubs or onto chairs to reach high objects. 6.In attempts to determine the daily exercise tolerance of a male COPD patient, all of the following questions might be asked EXCEPT A. How far can you walk your pet dog around the yard? B. Are you able to shave every day? C. Is your wife or a relative able to drive you to the grocery store? D. How many flights of stairs can you climb before you have to stop? C. A patient's exercise tolerance is not evaluated by determining if a relative can drive the patient somewhere. Exercise tolerance can be assessed by determining the patient's ease or difficulty in performing specific tasks. 7.The main goal of a pulmonary rehabilitation program should be to A. Reduce the amount of sputum coughed out every day B. Return the patient to his or her highest possible level of functioning C. Reduce the amount of D. Increase the patient's appetite to achieve weight gain B. Restoring the patient to his or her highest possible functional capacity is the main goal of a rehabilitation program. All other goals are secondary to this. 8.Your patient and her husband are both smokers. She is about to be discharged after treatment for bronchitis. Which of the following would you recommend? 1. She should stop smoking.

2. She should see her physician to get help to stop smoking. 3. Her husband should stop smoking.

4. She should switch to her husband's brand of cigarettes to reduce the conflict between them. A. 1, 2 B. 3 C. 1, 2, 3 D. 1, 2, 3, 4 C. The patient and her husband must both stop smoking. If she is the only one to quit, she will continue to inhale secondhand smoke from her husband. Her physician should be involved in her care to offer medical support and recommend group support. 1.Which of the following should the respiratory therapist evaluate to determine whether a patient's chest tube is functioning properly and removing pleural air? A. Fluid is present in the

collection chamber. B. The vacuum level is set at 15 cm H2O. C. Air is bubbling in the waterseal chamber. D. Air is bubbling in the suction control chamber. C. If air is seen to bubble in the water-seal chamber, it is known that vacuum is applied to the drainage system and the patient has pleural air (pneumothorax) that is being removed. The presence of fluid in the collection chamber confirms that the patient has pleural fluid that is being removed. It does not confirm that pleural air is being removed. A vacuum level of 15 cm H2O is normal and should result in air bubbling in the suction control chamber. However, it does not confirm that pleural air is being removed. 2.Your patient is being mechanically ventilated when the nurse calls you to evaluate the patient's condition. You discover that her breath sounds are absent over the left lung field, the left-sided percussion note is hyperresonant, and peak airway pressures have increased from 40 to 65 cm H2O. What would you recommend? A. Place a pleural chest tube into the right side. B. Increase the VT to better inflate the atelectatic left lung. C. Change the mode to synchronous intermittent mechanical ventilation from assist/control. D. Place a pleural chest tube into the left side. D. The physical signs indicate that the patient has a left-sided pneumothorax; a pleural chest tube is indicated. Do not place a tube into the right pleural space. Increasing the VT or changing the mode would not correct the problem. 3.You notice that air is bubbling through the water seal of the patient's pleural drainage system when she coughs. This tells you that A. The vacuum has to be increased B. Air is still leaking through a tear in the lung C. The proper level of vacuum has been set D. There is a leak in the system B. An air leak through the water seal when the patient coughs indicates that the air is coming from the patient's lung. If a leak exists in the system, air will bubble through the water seal at all times. The vacuum level has no impact on the air leak from the patient. 4.After a sleep study has been performed, your patient is given a diagnosis of obstructive sleep apnea. His physician asks for your advice on the best method of management. You would recommend that A. The patient should use nasal CPAP when sleeping B. The patient should sleep with an oropharyngeal airway to keep the tongue forward in the mouth C. The patient should always sleep on his back D. A tracheostomy should be

performed, and the patient should be placed on a volume-cycled ventilator to sleep A. Nasal CPAP would be indicated for the management of a patient with obstructive sleep apnea. An oropharyngeal airway would be indicated only in an obtunded patient with an obstructed upper airway. For many patients, sleeping on the back worsens the obstruction. A tracheostomy and mechanical ventilation may be indicated in a patient with central sleep apnea if all other efforts have failed. 5.An adult patient with obstructive sleep apnea and frequent pulse oximetry desaturations is fitted with a nasal CPAP system. A pressure of 7 cm H2O is set. After the CPAP system is set up, the patient's SpO2 value stays above 90%. How should the results be interpreted? A. The CPAP system has corrected the patient's problem. B. Greater CPAP pressure is needed. C. Improved gas flow is needed through the CPAP system. D. The delivered oxygen percentage should be increased. A. A normal SpO2 value indicates that the CPAP system at 7 cm H2O pressure has corrected the patient's problem. There is no need to raise the CPAP level because the patient's pulse oximetry value is normal. There is no indication of inadequate flow through the CPAP system. There is no need to raise the delivered oxygen percentage because the saturation is within the normal range. 6.Your mechanically ventilated patient is

going to have a flexible fiberoptic bronchoscopy performed. Of what kinds of considerations must you be aware? 1. The VT must be monitored for a leak.

2. The inspiratory flow resistance will increase. 3. The inspiratory pressure will decrease. 4. The inspiratory pressure will increase. A. 1, 2 B. 3 C. 2, 3 D. 1, 2, 4 D. Performing a bronchoscopy on a ventilated patient necessitates the use of a special bronchoscopy adapter between the endotracheal tube and the circuit. The bronchoscope is placed through the adapter, which may result in a VT leak. In addition, the bronchoscope partially obstructs the endotracheal tube, which increases resistance and causes inspiratory pressure to increase. 7.During the transport of your patient with a pneumothorax, the chest tube drainage system is pulled off of the drainage tubing and cracked open. Your best response is to A. Clamp the tube near the patient's chest at once B. Hold the distal end of the tubing a few centimeters below the surface of a bottle of sterile water or saline C. Leave the tube open to the

atmosphere D. Have the patient perform the Valsalva maneuver until a new system can be set up B. The easiest way to seal the chest tube but allow air leaking from the patient to escape is to place the distal end of the tube under water. Clamping the tube does not allow any pleural air to escape. Leaving the tube open allows pleural air to escape but also allows room air to enter the pleural space. The Valsalva maneuver would require breathholding by the patient for an extended time and is not practical. (See Table 18-2 on page 564.) 8.When preparing to assist the physician with the cardioversion of a patient, you must check the following: 1. A strong R wave should be seen on the ECG monitor. 2. The charge level should be set as ordered. 3. The electric paddles should be kept clean for the best possible conduction. 4. Ensure that the ECG electrodes are attached properly. A. 1, 3 B. 1, 2, 4 C. 3, 4 D. All of the above B. Cardioversion requires a strong R wave on the ECG monitor, a proper electrical charge level, and proper placement of the ECG leads. The electrode paddles should be well covered with a conducting jelly to maximize the flow of current into the patient

without a burn. 9.A 6-year-old patient has aspirated a tooth that was dislodged during a sporting event. The chest x-ray film shows the tooth to be lodged in the right mainstem bronchus; the neck x-ray film is normal. What would you recommend as the best way to quickly remove the tooth? A. Flexible fiberoptic bronchoscopy (FFB) B. Positive expiratory pressure breathing (PEP) C. Rigid tube bronchoscopy D. Postural drainage therapy (PDT) C. Rigid tube bronchoscopy is preferred in the removal of a large foreign body from a large airway. FFB may be used with a small foreign body or one that is in a smaller airway. PEP and PDT are not likely to dislodge a large foreign body such as a tooth. PDT with percussion may dislodge a foreign body. (See Box 18-2 on page 549.) 10.A patient is referred for a sleep study. The attending physician wants to know which parameters are measured during the study. You would measure all of the following EXCEPT A. SpO2 B. ECG C. Inspiratory and expiratory breathing efforts D. ABG values D. ABGs are not monitored during a sleep study because it is an invasive procedure and the patient would awaken as the blood

sample is drawn. The other items are routinely monitored during a sleep study. 11.Your patient is performing an exercise test and has the following signs andsymptoms: systolic blood pressure of 260 mm Hg, cyanosis, headache, and dizziness. Which of the following would you recommend? A. Continue the test until the patient's respiratory exchange ratio reaches 1.1. B. Stop the test. C. Continue the test until the patient complains of shortness of breath. D. Continue the test at a lower work level. B. The test should be stopped because the patient's signs and symptoms indicate that he is not tolerating the procedure. A healthy person who does not have dangerous signs and symptoms may tolerate a respiratory exchange ratio of 1.1 for a short time; this patient should not be pushed to this point. This patient has several signs and symptoms showing intolerance and does not need to be pushed to complain of shortness of breath. While the patient may tolerate the stress test at a lower work level, it is safer to stop the test and evaluate the patient. 12.A patient is performing a stress test. Which of the following respiratory exchange ratio values would confirm that the patient has reached the anaerobic threshold? A. 0.8 B. 0.9 C. 1.0

D. 1.1 C. When indirect calorimetry is used to evaluate a patient's oxygen consumption and carbon dioxide production during a stress test, the anaerobic threshold is confirmed by a respiratory exchange ratio (R) of 1.0. See the calculation for question 4 of the Written Registry Exam Self-Study Questions. Before the R value reaches 1.0 the patient will dramatically increase his/her tidal volume and respiratory rate when the tissues are hypoxic and anaerobic metabolism occurs. 13.You are assisting a physician with a surgical tracheostomy procedure on a patient with an oral endotracheal tube. When should you withdraw the endotracheal tube? A. After the tracheostomy tube has been inserted B. After the cuff of the tracheostomy tube has been inflated C. As the tip of the tracheostomy tube is placed into the stoma D. Before the stoma is made C. The endotracheal tube should be removed from the patient as the tracheostomy tube is to be placed into the stoma. This ensures that the patient has a secure airway throughout the procedure. 14.A medical student is assigned to care for a patient with a smoking history and who is suspected of having emphysema. She asks for recommendations on what tests might be used to help determine the diagnosis. You would recommend all of the following EXCEPT A. Pulmonary function testing

B. Chest radiograph C. Chest ultrasound D. Arterial blood gases C. An ultrasound of the chest would not be helpful since sound waves do not reflect off of air-filled spaces such as the lungs. Pulmonary function tests would provide laboratory evidence of the patient's lung volumes and expiratory flow. A chest radiograph would illustrate if the lungs are hyperinflated. Arterial blood gases would show if the patient is hypoxemic and retaining carbon dioxide.
Drainage position for the posterior basal segments of both lower lobes.

Drainage position for the lateral basal segment of the right lower lobe. The same segment in the left lung would be drained by positioning the patient similarly on the right side

Drainage position for the anterior basal segment of the left lower lobe. The same segment in the right lung would be drained by positioning the patient similarly on the left side

Drainage position for the lateral and medial segments of the right middle lobe.

Drainage position for the superior segments of both lower lobes

Drainage position for the superior and inferior segments of the lingula.

Drainage position for the posterior segments of both upper lobes.

Drainage position for the apical segments of both upper lobes.

Drainage positions in infants. A, Apical segment of upper lobes. B, Posterior segments of upper lobes. C, Anterior segments of upper lobes. D-F, Superior segments of lower lobes. G and H, Anterior basal segments of both lower lobes (H on right and left sides). I, Segments of the right middle lobe and lingula (shown). J, Posterior basal segments of the lower lobes.

TABLE 3-3 Evaluation of Hypoxemia

Drainage position for the anterior segments of both upper lobes.

*Subtract 1 torr of O2 from limits of mild and moderate hypoxemia for each year older than 60. A PaO2 value of <40 torr indicates severe hypoxemia in any patient at any age. TABLE 3-2 Age-Based Acceptable Levels of Partial Pressure of Oxygen in Arterial Blood (PaO2) When Breathing Room Air (21% Oxygen) at Sea Level

You might also like